Google Groups no longer supports new Usenet posts or subscriptions. Historical content remains viewable.
Dismiss

Aufklärung über Cantors Theorie

1,051 views
Skip to first unread message

Ganzhinterseher

unread,
Jul 17, 2022, 8:58:48 AM7/17/22
to
Drei Hinweise für Kinder, die die grundlegendsten Teile der Mengenlehre schon kennen.

(1)
Jede Menge positiver gerader natürlicher Zahlen enthält eine natürliche Zahl, die größer als ihre Anzahl ist. Das gilt für alle diese Mengen, also für unendlich viele, also auch für die unendliche Menge aller positiven geraden natürlichen Zahlen.

(2)
Auf der reellen Achse findet sich zwischen zwei beliebigen irrationalen Zahlen immer eine rationale Zahl. Also ist die Anzahl der irrationalen Zahlen nicht größer als die der rationalen Zahlen.

(3)
Für jede natürliche Zahl n findet sich ein Endsegment, E(n+1) = {n+1, n+2, n+3, ...} der natürlichen Zahlen, das n nicht enthält. Daraus schließen manche, dass der Schnitt aller Endsegmente leer ist. Sie übersehen oder verdrängen aber, dass jedes zum Beweis herangezogene Endsegment E(n+1) unendlich viele Zahlen enthält, so dass der Schnitt in keinem der unendlich viele Fälle leer ist.
Für Fortgeschrittene: Käme in ∩{E(k) : k ∈ ℕ} = { } kein einziges Endsegment vor, das in
∀k ∈ ℕ: ∩{E(1), E(2), ..., E(k)} = E(k) /\ |E(k)| = ℵ₀ (*)
fehlt, so könnte der Schnitt nicht leer sein. Alle Endsegmente aus (*) reichen jedenfalls nicht aus, den leeren Schnitt zu produzieren.

Gruß, WM

paule32

unread,
Jul 17, 2022, 10:58:02 AM7/17/22
to
Am 17.07.2022 um 14:58 schrieb Ganzhinterseher:
> ∀k ∈ ℕ: ∩{E(1), E(2), ..., E(k)} = E(k) /\ |E(k)| = ℵ₀ (*)
> fehlt, so könnte der Schnitt nicht leer sein.

schön....
säuberlich....
provokativ....

eben: gewiefter alter Kater : )

In Reinschrift:
Alle k sind Element von |N, wobei gilt:
die Vereinigung aller Endsegmente {E(1), ..., E(k)}
ergeben die Summe = E(K) - oder -
Alle negativen und positiven Summen - hier E(K) -
ergeben aleph_0.

Nun kann man streiten:
1. die Summe, also die Anzahl/Mächtigkeit der Mengen
oder auch hier als Endsegmente verdeutlicht, ist
ja mit dieser Definition gegeben, und steht auf
weisen Blatt Pappier ja mit schwarzer Farbe darauf.
Also könnte man argumentieren, das E(K) := 1 ist.

2. Da sich aber die Betrachtung hier auf "unendliche"
Mengen bezieht, wissen wir nicht, wie Groß diese
eine E(K) ist, und würden naiv sagen E(K) := ( ).
Also, dass ein leerer Schnitt vorliegt.

3. Dies führt unweigerlich zu einen Widerspruch bzw.
zu einer Mehrdeutigkeit.
Und Mehrdeutigkeiten sind ja in der Mathematik ein
Gestrüp in oder auf der Birne, was da nichts zu
suchen hat.

4. Also würden dann die MEhrheiten sagen "es kann 0
UND: "nicht dazu gehören".
(Also schon 2 Sachverhalte, die dann mit hessischen
süßen Senf den kleinen armen unbedarften Kindern
auf die Nase gestrichen wird, um die Mücken abzuhalten).

5. Also schon eine äußerst Ernste Zermello Angelegenheit,
wenn man versucht, in die tiefen der Mengen zu tauchen,
um dort Brotkrummen zu phischen (Sprich: Das Wort im
Munde verdrehend wohlende Leute mit Halbwissen zu
entziffern).

Ja, Ja, ...

IHR seht schon ....

Ein gewiefter "alter" (schwarzer) Karter namens Wolfgang.

Mit guten Kaffee-Spruch und pröstjen
der paule32 ...

P.S.: ... Der gerade etwas schmunzelt : ) ...

JVR

unread,
Jul 17, 2022, 12:44:22 PM7/17/22
to
Weitere einfache Verbesserungsmöglichkeiten im WM-Bot:
(1) Eine Eigenschaft, die für alle endlichen Untermengen einer Menge gilt, gilt nicht unbedingt für die Menge selber.
(2) Aussagen, die bekannten Tatsachen widersprechen, sollten vermieden werden. Niemand wird eine
Programmiergruppe an einer Fachhochschule ernst nehmen, die die einfachsten Grundlagen der Mathematik
nicht beherrscht.
(3) Komplexe Syntax und Semantik ist zu vermeiden. Das Ziel, den Gesprächspartner zu verwirren, kann man auch viel
weniger wortreich und mit einfachen Satzbau erreichen. Es genügen ein oder zwei undefinierte Begriffe.

Fritz Feldhase

unread,
Jul 17, 2022, 12:51:46 PM7/17/22
to
On Sunday, July 17, 2022 at 6:44:22 PM UTC+2, JVR wrote:
>
> Weitere einfache Verbesserungsmöglichkeiten im WM-Bot:
>
> (1) Eine Eigenschaft, die für alle endlichen Untermengen einer Menge gilt, gilt nicht unbedingt für die Menge selber.

Hmmm... Wie wär's da mit "ist endlich"? Nur mal so als ad hoc Idee in den Ring geworfen.

Das müsste einem Bot doch beizubringen sein, nein?

Stefan Schmitz

unread,
Jul 17, 2022, 12:57:31 PM7/17/22
to
Am 17.07.2022 um 18:44 schrieb JVR:

> Weitere einfache Verbesserungsmöglichkeiten im WM-Bot:
> (1) Eine Eigenschaft, die für alle endlichen Untermengen einer Menge gilt, gilt nicht unbedingt für die Menge selber.
> (2) Aussagen, die bekannten Tatsachen widersprechen, sollten vermieden werden. Niemand wird eine
> Programmiergruppe an einer Fachhochschule ernst nehmen, die die einfachsten Grundlagen der Mathematik
> nicht beherrscht.
> (3) Komplexe Syntax und Semantik ist zu vermeiden. Das Ziel, den Gesprächspartner zu verwirren, kann man auch viel
> weniger wortreich und mit einfachen Satzbau erreichen. Es genügen ein oder zwei undefinierte Begriffe.

Bei (3) würde ich Widerspruch anmelden.

Die Verwirrung funktioniert viel besser, wenn zusätzlich noch die
Formulierung verschleiert, was überhaupt ausgesagt wird. Außerdem würden
mit einfachen Sätzen Fehler viel leichter erkennbar.

Fritz Feldhase

unread,
Jul 17, 2022, 1:13:57 PM7/17/22
to
On Sunday, July 17, 2022 at 2:58:48 PM UTC+2, Ganzhinterseher wrote:

> Drei Hinweise für Kinder, die die grundlegendsten Teile der Mengenlehre schon kennen.
>
> (1)
> Jede Menge positiver gerader natürlicher Zahlen enthält eine natürliche Zahl, die größer als ihre Anzahl ist.

Da würdest Du den Kindern aber schon im ersten Satz Unsinn beibringen (wollen), Mückenheim.

Der Satz ist, so wie er dasteht, entweder unsinnig, oder falsch.

Richtig wäre:

"Jede ENDLICHE Menge positiver gerader natürlicher Zahlen enthält eine natürliche Zahl, die größer ist als die Anzahl der Elemente der Menge."

Im Falle einer UNENDLICHEN Menge wäre schon mal nicht klar, was hier mit der "Anzahl der Elemente der Menge" gemeint ist. Vermutluch -genaues weiß man leider nicht- ihre Kradinalzahl. Also im Falle einer UNENDLICHEN Menge positiver gerader natürlicher Zahlen die Kardinalzahl aleph_0.

In diesem Falle wäre die Behauptung falsch. Denn für keine natürliche Zahl n gilt n > aleph_0, daher auch für keine gerade natürliche Zahl.

> Das gilt für alle diese Mengen,

D a s? Schon wieder diese unsinnige Doppelung der Quantifizierung. Wie

* "Für alle natürliche Zahlen gilt An e IN: n + n = 2*n".

Typischer Mückenheim-Bot-Schwachsinn. Das sollten die Entwickler mal angehen und beseitigen!

Aber da es unendlich viele endliche Mengen natürlicher Zahlen gibt, stimmt jedenfalls

> für unendlich viele

ENDLICHE Mengen M positiver gerader natürlicher Zahlen gilt: M enthält eine natürliche Zahl, die größer ist als die Anzahl der Elemente der Menge.

> also auch für die unendliche Menge aller positiven geraden natürlichen Zahlen.

Wo das "also" herkommen soll, bleibt schleierhaft.

Alle ENDLICHEN Mengen M positiver gerader natürlicher Zahlen sind endlich. "Also" auch die unendliche Menge aller positiven geraden natürlichen Zahlen?!

Schönen Mist würden die Kinder da von Dir "lernen".

DARUM: Keine Bots im Schulunterricht!

Fritz Feldhase

unread,
Jul 17, 2022, 1:19:20 PM7/17/22
to
Ach so, Du meintest (1) als Ausdruck eines Sachverhalts, dem man dem Bot (in irgendeiner Form) beibringen müsste/sollte, ja natürlich. Z. B. in Bezug auf Mengen natürlicher Zahlen: "besitzt eine endliche Summe" (wobei die Summenbildung) sich auf die Elemente der Menge bezieht.

paule32

unread,
Jul 17, 2022, 1:40:44 PM7/17/22
to
Am 17.07.2022 um 18:51 schrieb Fritz Feldhase:
> Hmmm... Wie wär's da mit "ist endlich"? Nur mal so als ad hoc Idee in den Ring geworfen.
>
> Das müsste einem Bot doch beizubringen sein, nein?

Das machen/können nicht nur die WM-Bot's,
Das machen/können auch die Taschenrechner von Microsoft !
Oder wollen die nur den dummen Kunden haben, der den Mist
dann kauft und daran glaubt ?

Gebt doch mal unter Windows 11 (eines der fortschrittlichsten
Betriebssysteme) folgende Rechnung ein (im Wissenschaftlichen
Modus):

8 dividiert durch 3

was kommt raus ?


ja genau:

2,6666....67

hier wird audgerundet, weil der kleine Rechner ja mit KI
Software aka QBasic programmiert wurde !!!

Ja, jetzt im Ernst Zermello Verfahren .... wie die das
nur machen ?

Grübbel, Grübbel ....

Holladilölööle
Euer paule32

Fritz Feldhase

unread,
Jul 17, 2022, 1:51:25 PM7/17/22
to
On Sunday, July 17, 2022 at 2:58:48 PM UTC+2, Ganzhinterseher wrote:
>
> (2)
> Auf der reellen Achse findet sich zwischen zwei beliebigen irrationalen Zahlen immer eine rationale Zahl. Also ist die Anzahl der irrationalen Zahlen nicht größer als die der rationalen Zahlen.

Ja, das mag einem Kind (!) auf den ersten Blick plausibel vorkommen.

Falls es sich bei dem Kind um einen Mittelschüler handelt(e), könnte man es/ihn aber vermutlich mit folgendem "Gegenargument" einigermaßen verunsichern [wobei wir uns im folgenden auf positive Zahlen beschränken wollen]: Die rationalen Zahlen sind die Zahlen der Form n/m mit n,m e IN. Alle Zahlen der Form n/m * sqrt(2) sind irrational. Es gibt also offensichtlich mindestens so viele irrationale Zahlen wie rationale. Nun sind aber auch alle Zahlen der Form n/m * sqrt(3) irrational (wobei alle diese Zahlen von den Zahlen der Form n/m * sqrt(2) verschieden sind). Es gibt also mindestens "doppelt so viele" irrationale Zahlen wie rationale! Nun gilt aber für jede Primzahl p, dass die Zahlen der Form n/m * sqrt(p) irrational sind und sich von allen Zahlen der Form n/m * sqrt(q) - mit q prim und q =/= p - unterscheiden. Es gibt also offenbar unendlich-mal mehr irrationale Zahlen als rationale. D A S sollte ihm jedenfalls zu denken geben.

All diese Mengen irrationaler Zahlen sind "kammartig" ineinander "verschachtelt", wobei quasi jeweils die Zahlengerade mit allen rationalen Zahlen um den Faktor sqrt(2), sqrt(3), sqrt(5), ... nach rechts "gestreckt" ist, sodass die "Dichte" der irrationalen Zahlen doch deutlich höher sein muss - würde man meinen - als die der rationalen.

Nachdem man den Schüler auf diese Weise hinreichend verunsichert hat, könnte man ihm erklären, dass d a s allein aber nicht ausreicht, um auf eine größere Mächtigkeit der Menge der irrationalen Zahlen (als der der Menge der rationalen Zahlen) schließen zu können. [usw. usf.]

Jedenfalls glaube ich nicht, dass ihm d a n a ch das Mückenheimsche "Kammargument" noch als "stichhaltig" erschiene.

paule32

unread,
Jul 17, 2022, 2:02:37 PM7/17/22
to
Am 17.07.2022 um 19:51 schrieb Fritz Feldhase:
> Jedenfalls glaube ich nicht, dass ihm d a n a ch das Mückenheimsche "Kammargument" noch als "stichhaltig" erschiene.

braucht es doch nicht !
Wenn man langsam in die Thematik immer weiter fortschreitet ...
Wenn man dabei bedenkt, das nicht jeder Mittelschüler gleiche
kognitative Stärken wie sein am Nachbartisch sitzender hat ...

Manche Schüler begreifen durch lesen ...
Manche Schüler begreifen durch sehen ...
Manche Schüler brgreifen durch schreiben ...
Manche Schüler begreifen durch sprechen ...
Manche Schüler begreifen durch spielen ...

und gerade letzteres sollte gelten !

Was nützt es eine Scharr an Schülern zu haben, die ihre Klasse
nicht verteidigen - sprich zusammen halten, Probleme ansprechen,
Kommunikation betreiben (aber nicht beleidigend - eher mildernd)

Glauben kann man viel - ja.
Aber nur die Schüler, die sich untereinander stärken, sind dann
irgendwann (vielleicht nicht gleich und jetzt) die Helden von
morgen.

paule32

Ganzhinterseher

unread,
Jul 17, 2022, 3:33:45 PM7/17/22
to
JVR schrieb am Sonntag, 17. Juli 2022 um 18:44:22 UTC+2:

> (1) Eine Eigenschaft, die für alle endlichen Untermengen einer Menge gilt, gilt nicht unbedingt für die Menge selber.

Eine Eigenschaft, die für alle endlichen Anfangsabschnitte einer Menge gilt, gilt jedenfalls für den definierbaren Teil der Menge. Alle endlichen Anfangsabschnitte ergeben durch kaufmännische Addition die unendliche Menge, soweit sie definierbar ist.
Beispiel:
{1}
{1, 2}
{1, 2, 3}
{1, 2, 3, 4}
{1, 2, 3, 4, 5}
...
Kaufmännische Addition ergibt die Menge ℕ_def.

Der Glaube an eine größere Vereinigung ist ebenso wie der Glaube an ein höheres Wesen unwissenschaftliche Privatsache. Allerdings: Die Existenz eines höheren Wesens ist nicht auszuschließen. Eine größere Vereinigung aber mit Sicherheit.

> Programmiergruppe an einer Fachhochschule

Inzwischen Technische Hochschule Augsburg.

Gruß, WM

Ganzhinterseher

unread,
Jul 17, 2022, 3:35:54 PM7/17/22
to
Fritz Feldhase schrieb am Sonntag, 17. Juli 2022 um 18:51:46 UTC+2:
> On Sunday, July 17, 2022 at 6:44:22 PM UTC+2, JVR wrote:
> >
> > Weitere einfache Verbesserungsmöglichkeiten im WM-Bot:
> >
> > (1) Eine Eigenschaft, die für alle endlichen Untermengen einer Menge gilt, gilt nicht unbedingt für die Menge selber.
> Hmmm... Wie wär's da mit "ist endlich"? Nur mal so als ad hoc Idee in den Ring geworfen.
>
> Du glaubst an übernatürliche Kräfte. Unendlich ist eine Menge schon als Summe aller endlichen Anfangsabschnitte.

Beispiel:
{1}
{1, 2}
{1, 2, 3}
{1, 2, 3, 4}
{1, 2, 3, 4, 5}
...
Kaufmännische Addition ergibt die Menge ℕ_def.

Der Glaube an eine größere Vereinigung ist ebenso wie der Glaube an ein höheres Wesen unwissenschaftliche Privatsache. Allerdings: Die Existenz eines höheren Wesens ist nicht auszuschließen. Eine größere Vereinigung aber mit Sicherheit.

Gruß, WM

Ganzhinterseher

unread,
Jul 17, 2022, 3:37:34 PM7/17/22
to
Fritz Feldhase schrieb am Sonntag, 17. Juli 2022 um 19:13:57 UTC+2:
> On Sunday, July 17, 2022 at 2:58:48 PM UTC+2, Ganzhinterseher wrote:
> > Drei Hinweise für Kinder, die die grundlegendsten Teile der Mengenlehre schon kennen.
> >
> > (1)
> > Jede Menge positiver gerader natürlicher Zahlen enthält eine natürliche Zahl, die größer als ihre Anzahl ist.
> Da würdest Du den Kindern aber schon im ersten Satz Unsinn beibringen (wollen),

Nein, die Extrapolation ist gerechtfertigt. Dein Glaube ist nicht akzeptabel.

Gruß, WM

Tom Bola

unread,
Jul 17, 2022, 4:07:29 PM7/17/22
to
Fritz Feldhase schrieb:
> WM wrote:
>...

>> Jede Menge positiver gerader natürlicher Zahlen
>> enthält eine natürliche Zahl,
>> die größer als ihre Anzahl ist.

> Da würdest Du den Kindern aber schon im ersten Satz Unsinn
> beibringen (wollen), Mückenheim.
>
> Der Satz ist, so wie er dasteht, entweder unsinnig, oder falsch.
>
> Richtig wäre:
> "Jede ENDLICHE Menge positiver gerader natürlicher Zahlen enthält eine
> natürliche Zahl, die größer ist als die Anzahl der Elemente der Menge."

Hallo! Ja, und!?

Das ist eine Trivia, die v ö l l i g an dem vorbeigeht, das WM da faselt!

WM bringt die *Mächtigkeit* von Mengen mit der *Art* der Elemente in
einen totalverblödeten Zusammenhang.

Aber du übersiehst das und machst mit der Trivia weiter, die *nichts*
damit zu tun hat, was WM zum hundertausendsten Mal daher faselt.

Und im folgenden faselst auch du dann "stundenlang" irgendein verblödetes
Wenn und Aber, wobeii du die Trivia, "ernsthaft" untersuchst, nicht aber
das was WM daraus *für a l l e Mengen* behauptet!

Unglaublich!
Was du offenbar gar nicht mehr liest und raffst ist das:

A = { 1,2,4,6 } enthält 4 Elemente und fertig.

B = { 0, 1000^1000^1000, omega, aleph_42}
enthält ebenso 4 Elemente und fertig.

Nun, w a s hat die A r t der Elemente damit zu tun,
dass jedes Element die Mächtigkeit 1 hat, also so wie
auch aleph_42, das sicherlich grösser ist als 3.

Deshalb gibt es an dem oberen Stuss von WM nichts
mehr, stundenlang, zu untersuchen!

Du solltest wirklich mal zu einem Neurologen gehen (wegen
Demenzverdacht), da du nicht mal mehr das obige überblickst.

Oder kauf dir einen Hund oder was -- aber behalt deinen Stuss
in deinem Hirn drin...

JVR

unread,
Jul 17, 2022, 4:12:17 PM7/17/22
to
Es ist eine sehr wichtige Eigenschaft eines WM-ELIZA-Programmes - wie eines jeden Programmes -
den Unterschied zwischen einer Sache und dem Namen der Sache zu erkennen. Ist es ein Wert oder
ist es eine Referenz? Bezieht sich die Referenz auf einen Wert oder auf eine weitere Referenz?

Also nennen sich die Fachhochschulen nun Technische Hochschulen und die Technischen
Hochschulen nennen sich Technische Universitäten. Ja und? Was ändert sich dadurch?
Diese Art Hochstaplerei ist viel zu durchsichtig um jemanden zu täuschen. Schmalspur
bleibt Schmalspur und Mücke bleibt unser Usenet-Prefosser.

Tom Bola

unread,
Jul 17, 2022, 4:34:23 PM7/17/22
to
Nachtrag:

> Und im folgenden faselst auch du dann "stundenlang" irgendein verblödetes
> Wenn und Aber, wobeii du die Trivia "ernsthaft" untersuchst, nicht aber
> das was WM daraus *für a l l e Mengen* behauptet!

Nämlich dass die Tatsache, dass A = { 1, 2, 4, 6, ..., n } eine
Zahl n enthält, die grösser ist als die Mächtikeit |A| der Menge,

ein Beweis dafür sei, dass die Mengenlehre widerlegt ist!

Und Amicus labert daraufhin stundenlang über ganze Zahlen...
Message has been deleted

Ulrich D i e z

unread,
Jul 17, 2022, 5:44:47 PM7/17/22
to
Ganzhinterseher schrieb:

> Drei Hinweise für Kinder, die die grundlegendsten Teile der Mengenlehre schon kennen.
>
> (1)
> Jede Menge positiver gerader natürlicher Zahlen enthält eine natürliche Zahl, die größer als ihre Anzahl ist. Das gilt für alle diese Mengen, also für unendlich viele,

Es handelt sich da um unendlich viele Mengen von denen jede die
Eigenschaft hat, aus einer endlichen Anzahl an Elementen zu bestehen.

> also auch für die unendliche Menge aller positiven geraden natürlichen Zahlen.

Die jetzt in die Betrachtung einbezogene "unendliche Menge" hat diese
Eigenschaft nicht. Sie hat die diese Eigenschaft ausschliessende
Eigenschaft, aus unendlich vielen Elementen zu bestehen. Die jetzt in
die Betrachtung einbezogene Menge hat andere Eigenschaften als die zuvor
betrachteten Mengen.

Woher nimmst Du die Gewissheit, dass der Schluss, es gäbe in der
betreffenden Menge eine natürliche Zahl, die größer als ihre Anzahl ist,
auch bei Mengen richtig ist, die aus unendlich vielen Elementen bestehen?

Anders gefragt: Woher nimmst Du die Gewissheit, dass bei Mengen gerader
natürlicher Zahlen die Bedingung, aus einer endlichen Anzahl an
Elementen zu bestehen, nicht notwendig ist dafür, (mindestens) eine Zahl
zu enthalten, die größer ist als die Anzahl an in der Menge enthaltenen
Elementen?

> (2)
> Auf der reellen Achse findet sich zwischen zwei beliebigen irrationalen Zahlen immer eine rationale Zahl. Also ist die Anzahl der irrationalen Zahlen nicht größer als die der rationalen Zahlen.

Besagte beliebige irrationale Zahlen seien der Größe nach verschieden.
Sei A die kleinere, B die größere dieser beiden irrationalen Zahlen.
Es befindet sich nicht nur eine rationale und auch nicht nur eine
irrationale Zahl zwischen A und B.

> (3)
> Für jede natürliche Zahl n findet sich ein Endsegment, E(n+1) = {n+1, n+2, n+3, ...} der natürlichen Zahlen, das n nicht enthält. Daraus schließen manche, dass der Schnitt aller Endsegmente leer ist. Sie übersehen oder verdrängen aber, dass jedes zum Beweis herangezogene Endsegment E(n+1) unendlich viele Zahlen enthält, so dass der Schnitt in keinem der unendlich viele Fälle leer ist.

Es reicht nicht, _irgendwelche_ Zusammenstellungen mit endlich vielen
Endsegmenten "heranzuziehen", von denen jedes unendlich viele Elemente
enthält, sondern man man muss, ausgehend von n=m, _alle_ Endesgmente
E(k) für den Beweis "heranziehen", bei denen k > m ist, und das sind
unendlich viele Endsegmente.

Für k >=0 enthält jedes Endsegment E(k) eine natürliche Zahl k+1, die
die Eigenschaft hat, im Endsegment E(k+1) nicht enthalten zu sein und
von daher auch nicht in der Schnittmenge aller Endsegmente enthalten zu
sein. Somit hat jede natürliche Zahl die Eigenschaft, nicht in der
Schnittmenge _aller_ Endsegmente enthalten zu sein. Somit enthält die
Schnittmenge _aller_ Endsegmente keine natürlichen Zahlen. Da es sich bei
den Elementen der Mengen, aus denen der Schnitt gebildet wird, nur um
natürliche Zahlen handelt, ist die Schnittmenge aller Endsegmente somit
leer.

Mengen, die Endsegmente darstellen, haben unendlich viele Elemente.
Mengen, die Endsegmente darstellen und unendlich viele Elemente haben,
sind allesamt etwas anderes als die Schnittmenge aller dieser Mengen.
Die Schnittmenge aller dieser Mengen ist selbst kein Endsegment, denn
sie ist leer, was die Eigenschaft, unendlich viele Elemente zu haben,
ausschliesst, was wiederum ausschliesst, Endsegment zu sein.

Mit freundlichem Gruß

Ulrich

Fritz Feldhase

unread,
Jul 17, 2022, 5:48:58 PM7/17/22
to
On Sunday, July 17, 2022 at 10:07:29 PM UTC+2, Tom Bola wrote:
> Fritz Feldhase schrieb:
>
> A = { 1,2,4,6 } enthält 4 Elemente und fertig.

Schon, aber ist A eine Menge _gerader_ natürlicher Zahlen, ja?

Ist 1 eine gerade natürliche Zahl, bist Du Dir da wirklich sicher?

Du verstehst: Es ging um WMs Ausage

>> Jede Menge positiver ___gerader___ natürlicher Zahlen
>> enthält eine natürliche Zahl, die größer als ihre Anzahl ist.

Die Menge {1 ,2, 3} enthält z. B. keine solche Zahl.

Wohl aber die Menge {2, 4, 6}.

Erkennst Du den Unterschied?

Ich finde, manchmal schießt Du mit Deiner "Kritik" ein wenig über das Ziel hinaus - warum auch immer.

Mathematik scheint jedenfalls nicht Dein Ding zu sein. Dazu laberst Du zuviel Stuss.

> B = { 0, 1000^1000^1000, omega, aleph_42}
> enthält ebenso 4 Elemente und fertig.

Durchaus, nur was hat B mit WMs Aussage

>> Jede Menge positiver ___gerader natürlicher___ Zahlen
>> enthält eine natürliche Zahl, die größer als ihre Anzahl ist.

zu tun. Kannst Du mir das sagen?

Hinweis: Nichts.

Auf den Rest des Stussen einzugehen, den Du schreibst, lohnt sich nicht.

Aus meiner Sicht bis Du ein Fall für den Psychiater.

EOD

P.S. Bitte geh' mir aus der Sonne, ok?

Ulrich D i e z

unread,
Jul 17, 2022, 5:49:47 PM7/17/22
to
Ganzhinterseher schrieb:

> Der Glaube an eine größere Vereinigung ist ebenso wie der Glaube an ein höheres Wesen unwissenschaftliche Privatsache. Allerdings: Die Existenz eines höheren Wesens ist nicht auszuschließen. Eine größere Vereinigung aber mit Sicherheit.

Oft ist das Ganze mehr als nur die Summe seiner Teile. ;->

Mit freundlichem Gruß

Ulrich

Fritz Feldhase

unread,
Jul 17, 2022, 7:38:40 PM7/17/22
to
On Sunday, July 17, 2022 at 9:37:34 PM UTC+2, Ganzhinterseher wrote:
> Fritz Feldhase schrieb am Sonntag, 17. Juli 2022 um 19:13:57 UTC+2:
> > On Sunday, July 17, 2022 at 2:58:48 PM UTC+2, Ganzhinterseher wrote:
> > >
> > > Drei Hinweise für Kinder, die die grundlegendsten Teile der Mengenlehre schon kennen.
> > >
> > > (1)
> > > Jede Menge positiver gerader natürlicher Zahlen enthält eine natürliche Zahl, die größer als ihre Anzahl ist.
> > >
> > Da würdest Du den Kindern aber schon im ersten Satz Unsinn beibringen (wollen),
> >
> Nein, <blubber>

Doch, doch. Ja, ok, viell. hätte ich statt "Unsinn" "saudummen Scheißdreck" schreiben sollen.

Welche natürliche Zahl in der Menge aller positiven geraden natürlichen Zahlen ist denn größer als die "Anzahl" der Elemente dieser Menge?

Kann es so eine Zahl überhaupt geben?

Tipp: Sei WM e IN so eine Zahl. Die Menge aller positiven geraden natürlichen Zahlen ist aber unendlich, also muss sie mehr als WM Elemente enthalten. Widerspruch!

Also doch besser so:

> > Da würdest Du den Kindern aber schon im ersten Satz saudummen Scheißdreck vermitteln (wollen),

Tom Bola

unread,
Jul 17, 2022, 7:42:18 PM7/17/22
to
Fritz Feldhase schrieb:

> On Sunday, July 17, 2022 at 10:07:29 PM UTC+2, Tom Bola wrote:
>> Fritz Feldhase schrieb:
>>
>> A = { 1,2,4,6 } enthält 4 Elemente und fertig.
>
> Schon, aber ist A eine Menge _gerader_ matürlicher Zahlen, ja?

Auch der Typo tut eben nichts zur Sache! DAS ist eben der Punkt:

> Und im folgenden faselst auch du dann "stundenlang" irgendein verblödetes
> Wenn und Aber, wobei du die Trivia "ernsthaft" untersuchst, nicht aber
> das was WM daraus *für a l l e Mengen* behauptet!

Nämlich dass die Tatsache, dass A = { 2, 4, 6, ..., n } eine
Zahl n enthält, die grösser ist als die Mächtikeit |A| der Menge,

ein Beweis dafür sei, dass die Mengenlehre widerlegt ist!

Depp!

> Es ging um WMs Ausage
>
>>> Jede Menge positiver ___gerader___ natürlicher Zahlen
>>> enthält eine natürliche Zahl, die größer als ihre Anzahl ist.

Eben, nochmal:

Es geht WM darum, dass die Tatsache, dass A = { 2, 4, 6, ..., n }
eine Zahl n enthält, die grösser ist als die Mächtikeit |A| dieser
Menge,

ein Beweis dafür sei, dass die Mengenlehre widerlegt ist!

Aber du Blödmann schreibst cocksuckst jeden Tag WM pausenlos mit
deinem Scheiss über gerade Zahlen, währen WM in dir Idioten ein
Forum dafür findet, dass er die Mengenlehre widerlegt hat!

Weshalb? Na weil:

Jede Menge positiver gerader natürlicher Zahlen enthält eine
natürliche Zahl, die größer als ihre Anzahl ist.

Darüber oder ähnliches im Rahmen der Widerlegung der Mengenlehre
werdet
ihr beiden Kollegen im Geiste WM und du Maniker

auch morgen wieder stundelang den Scheissdreck aus euren Hirnmissbildungen
in die grosse, weite Welt posaunen.

Du gehörst in die Klappse und was du hier treibst ist widerlich krank.

Tom Bola

unread,
Jul 17, 2022, 8:14:23 PM7/17/22
to
Fritz Feldhase schrieb:

> On Sunday, July 17, 2022 at 10:07:29 PM UTC+2, Tom Bola wrote:
>> Fritz Feldhase schrieb:

> Durchaus, nur was hat B mit WMs Aussage
>
>>> Jede Menge positiver ___gerader natürlicher___ Zahlen
>>> enthält eine natürliche Zahl, die größer als ihre Anzahl ist.
>
> zu tun. Kannst Du mir das sagen?
>
> Hinweis: Nichts.

Ja.

A = { 0,2,4,6 } enthält 4 Elemente und fertig.

B = { 0, 1000^1000^1000, omega, aleph_42}
enthält ebenso 4 Elemente und fertig.

|A| = |B| = 4
Maximum(A) = 6 > 4
Maximum(B) = aleph_42 > 4

Also, w a s hat die A r t der Elemente damit zu tun,
dass jedes Element die Mächtigkeit 1 hat, egal wie gross
sein Zahlenwert ist.

Nichts, auch wenn WM das als "Beweis" absondert.

Die Mengen der Mengenlehre bestehen eben nicht nur
aus Elementen "___gerader natürlicher___" Zahlen.


Aber soweit reicht dein Horizont nicht, du quatschst tagelang
ausführlichst über alles, was WM dir vor die Beine wirft,
wie es sich für einen Psycho deiner Sorte eben gehört.

Und deine "EOD" sind einen Dreck wert, WM hast du das schon,
ROFL, oft mehrmals am Tag "angedroht", auch bei den Amis...

Du bist *schon deshalb* jemand, dessen Gerede einen Dreck wert ist.

Im übrigen habe ich nicht mit dir diskutiert, sondern dir, wie du
selbst sagst, eine "Kritik" geschenkt.

Ob du darauf "wirklich nicht" antwortest, LOL, ist scheissegal,
das alles zeigt nur wie krank du Komiker bist.

Juergen Ilse

unread,
Jul 17, 2022, 8:39:36 PM7/17/22
to
Hallo,

Ganzhinterseher <askas...@gmail.com> wrote:
> Drei Hinweise für Kinder, die die grundlegendsten Teile der Mengenlehre schon kennen.
>
> (1)
> Jede Menge positiver gerader natürlicher Zahlen enthält eine natürliche Zahl, die größer als ihre Anzahl ist. Das gilt für alle diese Mengen, also für unendlich viele, also auch für die unendliche Menge aller positiven geraden natürlichen Zahlen.

Solange da die leere Menge nicht ausgeschlossen ist, ist der Satz flacsh ...

> (2)
> Auf der reellen Achse findet sich zwischen zwei beliebigen irrationalen Zahlen immer eine rationale Zahl. Also ist die Anzahl der irrationalen Zahlen nicht größer als die der rationalen Zahlen.

Flacshe Schlussfolgerung. Das sowohl die rationalen Zahlen in den rellen
Zalen als auch die reellen Zahlen in den rationalen Zahlen "dicht liegen,
besagt ueber die Maechtigkeiten der rationalen Zahlen und der rellen Zahlen
nur aus, dass beide Mengen unendlich sind, mehr nicht.

> (3)
> Für jede natürliche Zahl n findet sich ein Endsegment, E(n+1) = {n+1, n+2, n+3, ...} der natürlichen Zahlen, das n nicht enthält. Daraus schließen manche, dass der Schnitt aller Endsegmente leer ist.

Diese Schlussfolgerung ist ja auch richtig.

> Sie übersehen oder verdrängen aber, dass jedes zum Beweis herangezogene Endsegment E(n+1) unendlich viele Zahlen enthält, so dass der Schnitt in keinem der unendlich viele Fälle leer ist.

Nein, das wird *nicht* uebersehen, weil es irrelevant ist. Wenn es zu
jeder natuerlichen Zahl n ein Endsegment gibt, dass n *nicht* entahelt,
dann muss der Schnitt *aller* Endsegmente leer sein. Das folgt aus der
Definition des Schnittes (die herr Mueckeheim offenbar auch nicht be-
griffen hat).

Tscuhess,
Juergen Ilse (jue...@usenet-verwaltung.de)

Juergen Ilse

unread,
Jul 17, 2022, 8:57:00 PM7/17/22
to
Hallo,

paule32 <paule...@gmail.com> wrote:
> Das machen/können auch die Taschenrechner von Microsoft !
> Oder wollen die nur den dummen Kunden haben, der den Mist
> dann kauft und daran glaubt ?
>
> Gebt doch mal unter Windows 11 (eines der fortschrittlichsten
> Betriebssysteme) folgende Rechnung ein (im Wissenschaftlichen
> Modus):
>
> 8 dividiert durch 3
>
> was kommt raus ?
>
>
> ja genau:
>
> 2,6666....67
>
> hier wird audgerundet, weil der kleine Rechner ja mit KI
> Software aka QBasic programmiert wurde !!!

Das ist fuer die meisten Taschhenrechner normal, da die meisten Taschenrechner,
fuer die die Bezeichnung "wiseenschhhaftlich" nichht voellig zu Unrecht
tragen, intern eine hhoehere Rechengenauigkeit aufweisen als sie anzeigen
und eigentlich *immer* den Anzeigewert entsprechend runden falls noetig.

Tschuess,
Juergen Ilse (jue...@usenet-verwaltung.de)

Tom Bola

unread,
Jul 17, 2022, 9:03:32 PM7/17/22
to
Fritz Feldhase schrieb:
> On Sunday, July 17, 2022 at 9:37:34 PM UTC+2, Ganzhinterseher wrote:
>> Fritz Feldhase schrieb am Sonntag, 17. Juli 2022 um 19:13:57 UTC+2:
>>> On Sunday, July 17, 2022 at 2:58:48 PM UTC+2, Ganzhinterseher wrote:
>>>>
>>>> Drei Hinweise für Kinder, die die grundlegendsten Teile der Mengenlehre schon kennen.
>>>> (1) Jede Menge positiver gerader natürlicher Zahlen enthält eine natürliche Zahl, die größer als ihre Anzahl ist.
>>>>
>>> Da würdest Du den Kindern aber schon im ersten Satz Unsinn beibringen (wollen),
>>>
>> Nein, <blubber>
>
> Doch, doch. Ja, ok, viell. hätte ich statt "Unsinn" "saudummen Scheißdreck" schreiben sollen.
>
> Welche natürliche Zahl in der Menge aller positiven geraden natürlichen Zahlen ist denn größer als die "Anzahl" der Elemente dieser Menge?

Worauf das immer hinausläuft:

☆ "Es gibt" nur soviele Indizes wie Elementarpartikel im Universum,
oder derartiges (Atome oder was).

☆ Deshalb "gibt es" nicht genügend Indizes für ("aktual") unendlich
viele (natürliche) Zahlen.

☆ Wenn "bereits" die grösste gerade natürliche Zahl in der Menge
der geraden natürlichen Zahlen mehr als doppelt so gross ist
wie der
grösste mögliche Index (sogar "aktual" gesehen), dann
muss dieses Maximum also doppelt so unmöglich sein, wie
*ihr* "letzter" Index, von welchem WM *immer* ausgeht.

☆ Das widerlegt laut WM a) mittels der "Argumente" der maximalen Anzahl
der "aktual möglichen" ungefähr 10^80 Atome und damit Indizes, und b)
s o g a r rein logisch ebenfalls die "potenziell" maximal möglichen
Zahl dieser Indizes sogar etwa *allein* schon mit dem Beispiel der
Menge der geraden natürlichen Zahlen um den Faktor zwei.
Natürlich lassen sich noch weit "schlimmere" Beispiele finden,
zBl. die Menge der Kubikzahlen, oder der Fakultäten von "allen" n.

☆ WM's Punkt ist bei diesem Beispiel aber, wie der eben gesagt hat,
dass dieses Beispiel der Menge der geraden natürlichen Zahlen
ganz besonders "kinderleicht" zu verstehen ist!

Und zwar als Beispiel dafür dass die Menge kleiner ist, als ihr Inhalt
direkt oder indirekt (über die Indizes) "erlaubt".

Das obige wird WM natürlich in irgendeiner Form in seine "Fachliteratur"
einarbeiten, um damit Neulinge erfolgreich und nachhaltig zu verblöden.

Und das ist das, was du, also das Fritsche, mehrfach täglich machst,
nämlich WM willig zuzuarbeiten für seine baldigen Veröffentlichungen.

Das ist entsetzlich, dass der soviel Methodik und Fachwissen für sein
Verblödungswerk TÄGLICH, auf sein Geheiss hin, automatisch und frei
Haus von solchen Arschlöchern bekommt wie dir.

Juergen Ilse

unread,
Jul 17, 2022, 9:04:07 PM7/17/22
to
Hallo,

Fritz Feldhase <franz.fri...@gmail.com> wrote:
> Richtig wäre:
>
> "Jede ENDLICHE Menge positiver gerader natürlicher Zahlen enthält eine natürliche Zahl, die größer ist als die Anzahl der Elemente der Menge."

Vorsichtshalber sollte man noch fordern, dass es eine "nichth leere Menge
positiver ganzer Zahlen" ist, denn die leere Menge erfuellt die Beehauptung
*nicht*. Eine Menge positiver ganzer Zahlen ist eine Menge, deren Elemente
alle positive ganze Zahlen sind. Diese Definition erfuellt auc die leere
Menge ...

> DARUM: Keine Bots im Schulunterricht!

Dem ist natuerlichh zuzustimmen.

Tschuess,
Juergen Ilse (jue...@usenet-verwaltung.de)

Juergen Ilse

unread,
Jul 17, 2022, 9:10:09 PM7/17/22
to
Hallo,

Fritz Feldhase <franz.fri...@gmail.com> wrote:
> On Sunday, July 17, 2022 at 10:07:29 PM UTC+2, Tom Bola wrote:
>> Fritz Feldhase schrieb:
>>
>> A = { 1,2,4,6 } enthält 4 Elemente und fertig.
>
> Schon, aber ist A eine Menge _gerader_ matürlicher Zahlen, ja?
>
> Ist 1 eine gerade natürliche Zahl, bist Du Dir da wirklich sicher?
>
> Du verstehst: Es ging um WMs Ausage
>
>>> Jede Menge positiver ___gerader___ natürlicher Zahlen
>>> enthält eine natürliche Zahl, die größer als ihre Anzahl ist.
>
> Die Menge {1 ,2, 3} enthält z. B. keine solche Zahl.

Da diese Menge mit der 1 und der 3 aber Zahlen enthaelt, die keine
"positiven geraden natuerlichen Zahlen" sind, ist es keine "Menge
positiver gerader natuerlichher Zahhlen" ...

Tschuess,
Juergen Ilse (jue...@usenet-verwaltung.de)

paule32

unread,
Jul 18, 2022, 5:23:58 AM7/18/22
to
Am 18.07.2022 um 02:56 schrieb Juergen Ilse:
> Das ist fuer die meisten Taschhenrechner normal, da die meisten Taschenrechner,
> fuer die die Bezeichnung "wiseenschhhaftlich" nichht voellig zu Unrecht
> tragen, intern eine hhoehere Rechengenauigkeit aufweisen als sie anzeigen
> und eigentlich*immer* den Anzeigewert entsprechend runden falls noetig.

ja.
nun stell Dir aber mal vor, das sich dadurch eine Abweichung von 1 %
ergibt - und das in kosmische Beträge ...

Da wird dann ein Winkel von 1 Grad schonmal schnell 10 Grad (jetzt auf
die Entfernung bezogen).

Ich weiß, das macht die Menschheit auch nicht mehr Fett.
Aber dafür wurden ja "geeichte" Geräte entworfen.

Gruß, paule32

Fritz Feldhase

unread,
Jul 18, 2022, 5:53:03 AM7/18/22
to
On Sunday, July 17, 2022 at 7:13:57 PM UTC+2, Fritz Feldhase wrote:
> On Sunday, July 17, 2022 at 2:58:48 PM UTC+2, Ganzhinterseher wrote:
> >
> > Drei Hinweise für Kinder, die die grundlegendsten Teile der Mengenlehre schon kennen.
> >
> > (1) Jede Menge positiver gerader natürlicher Zahlen enthält eine natürliche Zahl, die größer als ihre Anzahl ist.
> >
> Da würdest Du den Kindern aber schon im ersten Satz Unsinn beibringen (wollen), Mückenheim.
>
> Der Satz ist, so wie er dasteht, entweder unsinnig, oder falsch.
>
> Richtig wäre:

[nach einem Hinweis von JI korrigiert:]

"Jede nichtleere ENDLICHE Menge positiver gerader natürlicher Zahlen enthält eine natürliche Zahl, die größer ist als die Anzahl der Elemente der Menge."

Fritz Feldhase

unread,
Jul 18, 2022, 6:17:53 AM7/18/22
to
On Monday, July 18, 2022 at 3:10:09 AM UTC+2, Juergen Ilse wrote:
> Fritz Feldhase wrote:
> > On Sunday, July 17, 2022 at 10:07:29 PM UTC+2, Tom Bola wrote:
> > >
> > > A = {1, 2, 4, 6} enthält 4 Elemente und fertig.
> > >
> > > Schon, aber ist A eine Menge _gerader_ natürlicher Zahlen, ja?

Nein, und damit hat die Aussage nichts mit WMs Behauptung zu tun. Ebensogut könnte man hier "2 + 2 = 4" ins Feld führen.

> > Du verstehst: Es ging um WMs Aussage
> > > >
> > > > Jede Menge positiver ___gerader___ natürlicher Zahlen
> > > > enthält eine natürliche Zahl, die größer als ihre Anzahl ist.
> > > >
> > Die Menge {1, 2, 3} enthält z. B. keine solche Zahl.
> >
> Da diese Menge mit der 1 und der 3 aber Zahlen enthaelt, die keine
> "positiven geraden natuerlichen Zahlen" sind, ist es keine "Menge
> positiver gerader natuerlichher Zahlen" ...

Hat das jemand behauptet? Es ist ein Beispiel dafür, dass nicht jede (endliche) Menge positiver natürlicher Zahlen eine natürliche Zahl enthält, die größer ist als die Anzahl der Elemente der Menge. Das war Erklärung für Frau Tombola gedacht. Deinen Dünnpfiff brauchts jetzt nicht auch noch dazu. Heb Dir den für WM auf.

Tom Bola

unread,
Jul 18, 2022, 6:24:43 AM7/18/22
to
Juergen Ilse schrieb:
> positiver gerader natuerlichher Zahlen" ...

Nebenbei bemerkt enthält übrigens bereits jede Menge positiver
natürlicher Zahlen, die weder das Element 0 noch 1 enthält, also
A={2,3,...,n} eine natürliche Zahl n > |A|, die größer als ihre
Anzahl ist.
Beweis: Weil in Mengen jedes Element nur einmal enthalten ist (!)
und die *geordnete* Menge {2,3,...,n} mit Mächtigkeit M0=Min(|n+1|)
ein minimales n enthält, so dass alle weiteren ebenfalls geordneten
Möglichkeiten eine grösseren Mächtigkeit Mn > M0 haben...

Tom Bola

unread,
Jul 18, 2022, 6:28:31 AM7/18/22
to
Fritz Feldhase schrieb:

> On Monday, July 18, 2022 at 3:10:09 AM UTC+2, Juergen Ilse wrote:
>> Fritz Feldhase wrote:
>>> On Sunday, July 17, 2022 at 10:07:29 PM UTC+2, Tom Bola wrote:
>>> >
>>> > A = {1, 2, 4, 6} enthält 4 Elemente und fertig.
>>> >
>>> > Schon, aber ist A eine Menge _gerader_ natürlicher Zahlen, ja?
>
> Nein, und damit hat die Aussage nichts mit WMs Behauptung zu tun. Ebensogut könnte man hier "2 + 2 = 4" ins Feld führen.
>
>>> Du verstehst: Es ging um WMs Aussage
>>> > >
>>> > > Jede Menge positiver ___gerader___ natürlicher Zahlen
>>> > > enthält eine natürliche Zahl, die größer als ihre Anzahl ist.
>>> > >
>>> Die Menge {1, 2, 3} enthält z. B. keine solche Zahl.
>>>
>> Da diese Menge mit der 1 und der 3 aber Zahlen enthaelt, die keine
>> "positiven geraden natuerlichen Zahlen" sind, ist es keine "Menge
>> positiver gerader natuerlichher Zahlen" ...
>
> Hat das jemand behauptet?

Aber WM hat impliziert, dass das eine Besonderheit ist, die eine
Widerlegung der Mengenlehre mit sich bringen.

> Es ist ein Beispiel dafür, dass nicht jede (endliche) Menge positiver
> natürlicher Zahlen eine natürliche Zahl enthält, die größer ist als
> die Anzahl der Elemente der Menge.

Das ist keine "Kunst", es enthält zBl. bereits jede Menge positiver
natürlicher Zahlen, die weder das Element 0 noch 1 enthält, also
A={2,3,...,n} eine natürliche Zahl n > |A|, die größer als ihre
Anzahl ist.

Jetzt geh wieder zu dem Amis, du musst noch ~15 Stunden tippen...

Tom Bola

unread,
Jul 18, 2022, 6:35:07 AM7/18/22
to
Tom Bola schrieb:

> Aber WM hat impliziert, dass das eine Besonderheit ist, die eine
> Widerlegung der Mengenlehre mit sich bringen.

... hat eher impliziert, dass diese "grundlegende" Eigenschaft
"der Zahlen", die er als Platoniker wahrnimmt, deshalb, weil damit
auch die Zahlen jenseits über allem anderen im (auch materiellen)
Universum stehen, insbesondere also über jeder möglichen Logik,
die Widerlegung der Mengenlehre bedeutet...

Das Spinner-Hasi kapiert WMs Gedankenwelt gar nicht, und faselt
sein Zeug mithin jarhein, jahraus, round-clock an WM total vorbei...

Tsst... ;)

Tom Bola

unread,
Jul 18, 2022, 6:46:39 AM7/18/22
to
Tom Bola schrieb:

> Das ist keine "Kunst",

... sondern wegen der Unitarität jedes Mengenelements trivial,
und dazu muss man eben nicht, so wie WM, erst ein "Wunder" dafür
nennen, wie die ganzen Zahlen (oder gar die Quadratzahlen, oh),
dass eine Menge gewöhnlicher natürlicher Zahlen nicht nur
ebensoviele, sondern sogar grössere Elemente "zitiert",
als die Menge mächtig ist.

Hasi, du solltest dich lieber mal damit auseinandersetzen, wie
es möglich ist, dass deine Hirnmissbildung auf solch eine Trivia
stundenlang penibel eingeht.

Sowas macht nur jemand, der unterbelichtet oder krank ist...

> es enthält zBl. bereits jede Menge positiver
> natürlicher Zahlen, die weder das Element 0 noch 1 enthält, also
> A = { 2, 3, ..., n} eine natürliche Zahl n > |A|, die größer als
> ihre Anzahl ist.

Fritz Feldhase

unread,
Jul 18, 2022, 6:48:16 AM7/18/22
to
On Sunday, July 17, 2022 at 9:33:45 PM UTC+2, Ganzhinterseher wrote:
> JVR schrieb am Sonntag, 17. Juli 2022 um 18:44:22 UTC+2:
>
> > (1) Eine Eigenschaft, die für alle endlichen Untermengen einer Menge gilt, gilt nicht unbedingt für die Menge selber.
> >
> Eine Eigenschaft, die für alle endlichen Anfangsabschnitte [von IN] gilt, gilt

nicht notwendigerweise auch für IN selbst. "ist endlich" ist ein Beispiel für so eine "Eigenschaft".

Eine andere wäre "hat ein letztes/maximals Element".

Eine weitere: "ist ungleich IN".

Zweifellos ist jeder endliche Anfangsabschnitt von IN ungleich IN. IN ist aber nicht ungleich IN.

Dem widerpricht natürlich der berühmte Mückenschluss:

| Aus An e IN: Phi[{1, 2, ..., n}] folgt Phi[IN].

D a s beweist dann die Widersprüchlichkeit der Mengenlehre. Denn beim Mückenschluss handelt es sich um eine unumstößliche Wahrheit! (Daher ist er das quasi das "Grundaxiom" der sog. Mückenmatik.) Der Mückenschluss ergibt: Jeder endliche Anfangsabschnitt von IN ist ungleich IN, daher ist auch IN ungleich IN. Das ist natürlich ein Widerspruch. Daher muss die Prämisse dieses Schlusses, nämlich dass jeder endliche Anfangsabschnitt von IN ungleich IN ist, falsch sein! Mit anderen Worten, IN muss selbst ein endlicher Anfangsabschnitt von IN sein. Kurz: IN ist endlich!




Tom Bola

unread,
Jul 18, 2022, 6:57:05 AM7/18/22
to
Richtigstellung:

> ☆ "Es gibt" nur soviele Indizes wie Elementarpartikel im Universum,
> oder derartiges (Atome oder was).

Die ursprüngliche Schätzung bezog sich auf Fermionen, damals ~10^80.

"Elementarpartikel" war Unsinn, weil Bosonen, d.h. alles andere
als Fermionen, die in beliebiger Zahl "auftauchen" können...

Tom Bola

unread,
Jul 18, 2022, 6:58:55 AM7/18/22
to
Fritz Feldhase schrieb:
Bravo!

Fritz Feldhase

unread,
Jul 18, 2022, 7:19:00 AM7/18/22
to
On Monday, July 18, 2022 at 12:28:31 PM UTC+2, Tom Bola wrote:
> Fritz Feldhase äußerte:
> >
> > Es ist ein Beispiel dafür, dass nicht jede (endliche) Menge positiver
> > natürlicher Zahlen eine natürliche Zahl enthält, die größer ist als
> > die Anzahl der Elemente der Menge.
> >
> Das ist keine "Kunst", es enthält zBl. bereits jede Menge positiver
> natürlicher Zahlen, die weder das Element 0 noch 1 enthält, also
> A={2,3,...,n} eine natürliche Zahl n > |A|, die größer als ihre
> Anzahl ist.

Nö. Weder A = {}, noch A = {2, 3, 4, ...} enthält (enthalten) so eine Zahl.

Noch immer nicht genug Unsinn gelabert heute?

Tom Bola

unread,
Jul 18, 2022, 7:33:07 AM7/18/22
to
Fritz Feldhase schrieb:
Schaumerma, gemeint ist natürlich was da steht:

A = {2,3,...,n} *)

und gemeint ist ein festes n, weil da nicht steht für alle n in IN.
In weiteren Posts dazu habe ich auch immer *geordnete* Menge geschrieben,
so dass n hier *) das maximale Element ist.

So ist dann A = {2, 3, ..., 5} mit Max(A) = 5 und |A| = 4 < 5.

Und dass ein leeres A = {} nicht in das *GENANNTE* Schema A={2,3,...,n}
passt, das siehst du doch wohl noch!

Balla, balla, du bist wirklich ein Idiot...

Tom Bola

unread,
Jul 18, 2022, 7:39:51 AM7/18/22
to
Nachtrag:

> Schaumerma, gemeint ist natürlich was da steht:
>
> A = {2,3,...,n} *)

Ich gebe feierlich zu, dass die Schreibweise A = {2,3,...,N}
mit grossem N besser eingängig gewesen wäre.

Aber das ändert nichts daran, was da steht, und es muss allen
gesunden Leuten hier ein Rätsel sein, wie du dazu kommst *)
in deine *Wunschvorstellung* A = {2,3,n,...} abzuändern,
das ist *wirklich*, also wirklich, krank!

Bitte bestätige das.

Tom Bola

unread,
Jul 18, 2022, 7:50:29 AM7/18/22
to
Tom Bola schrieb:

> Balla, balla, du bist wirklich ein Idiot...

... keine Bange... ich ebenfalls...

Fritz Feldhase

unread,
Jul 18, 2022, 8:59:32 AM7/18/22
to
On Monday, July 18, 2022 at 1:39:51 PM UTC+2, Tom Bola wrote:
> Nachtrag:
>
> Ich gebe feierlich zu, dass die Schreibweise A = {2,3,...,N}
> mit grossem N besser eingängig gewesen wäre.

Ach, das ist es doch nicht mal.

Es geht mir darum:

Wenn Du schreibst

> "Es enthält z. B. bereits jede Menge positiver
> natürlicher Zahlen, die weder das Element 0 noch 1 enthält, also
> A={2,3,...,n} eine natürliche Zahl n > |A|, die größer als ihre
> Anzahl ist."

dann ist das in meinen Augen ein ziemliches Durcheinander. Ich lese das jedenfalls so:

> "Es enthält z. B. bereits jede Menge positiver
> natürlicher Zahlen, die weder das Element 0 noch 1 enthält, also z. B.
> A={2,3,...,n}, eine natürliche Zahl n > |A|, die größer als ihre
> Anzahl ist."

Es handelt sich dann also um die "generalisierte" (->Allquantor) Behauptung

"Es enthält z. B. bereits jede Menge positiver natürlicher Zahlen, die weder das Element 0 noch 1 enthält, eine natürliche Zahl, die größer als ihre Anzahl ist."

in die ein Beispiel, nämlich A = {2,3, ...,n} mit n e IN \ {0, 1}, - etwas unglücklich - "eingeflochten" ist.

Dass dem so ist, erhellt daraus, dass (mal von der Endlichkeit diese Menge abgesehen) {2,3,...,n} schon deshalb nicht als "schematischer Vertreter" für "_jede_ Menge positiver natürlicher Zahlen" stehen kann, weil nicht jede solche Menge eine 2 oder eine 3 enthalten muss (wenn sie denn überhaupt Elemente enthält).

In diesem konkreten Beispiel hast Du eine endliche Menge angeführt, ja, aber die eigentliche Aussage/Behauptung bezieht sich nicht auf dieses konkrete Beispiel, sonder auf "jede Menge positiver natürlicher Zahlen, die weder das Element 0 noch 1 enthält".

Wenn Du die Behauptung "enthält eine natürliche Zahl, die größer als ihre Anzahl ist" für "_alle_ endlichen (nichtleeren) Mengen positiver natürlicher Zahlen, die weder das Element 0 noch 1 enthalten", aufstellen willst, dann musst Du das auch entsprechend sagen/ausdrücken. Ebenso, wenn Du sie lediglich für Mengen der Form {2,3, ...,n} mit n e IN \ {0, 1} aufstellen willst. Beides "zusammen" schlägt sich leider.

Vielleicht wolltest Du "eigentlich" so was sagen/ausdrücken:

> "Es enthält z. B. bereits jede endliche (nichtleere) Menge positiver
> natürlicher Zahlen, die weder das Element 0 noch 1 enthält, also jede Menge
> A = {n_1, ... n_k} mit k e IN und n_i e IN \ {0} für alle i e IN, 1 <= i <= k,
> eine natürliche Zahl die größer als ihre Anzahl ist,
> also ein n_j mit j e IN und 1 <= j <= k, so dass n_j > |A| ist."

So und nun genug davon. EIN Mückemheim reicht mir.

EOD

Tom Bola

unread,
Jul 18, 2022, 9:28:22 AM7/18/22
to
Fritz Feldhase schrieb:

> On Monday, July 18, 2022 at 1:39:51 PM UTC+2, Tom Bola wrote:
>> Nachtrag:
>>
>> Ich gebe feierlich zu, dass die Schreibweise A = {2,3,...,N}
>> mit grossem N besser eingängig gewesen wäre.
>
> Ach, das ist es doch nicht mal.
>
> Es geht mir darum:
>
> Wenn Du schreibst
>
>> "Es enthält z. B. bereits jede Menge positiver
>> natürlicher Zahlen, die weder das Element 0 noch 1 enthält, also
>> A={2,3,...,n} eine natürliche Zahl n > |A|, die größer als ihre
>> Anzahl ist."
>
> dann ist das in meinen Augen ein ziemliches Durcheinander. Ich lese das jedenfalls so:
>
>> "Es enthält z. B. bereits jede Menge positiver
>> natürlicher Zahlen, die weder das Element 0 noch 1 enthält, also z. B.
>> A={2,3,...,n}, eine natürliche Zahl n > |A|, die größer als ihre
>> Anzahl ist."
>
> Es handelt sich dann also um die "generalisierte" (->Allquantor) Behauptung
>
> "Es enthält z. B. bereits jede Menge positiver natürlicher Zahlen, die weder das Element 0 noch 1 enthält, eine natürliche Zahl, die größer als ihre Anzahl ist."
>
> in die ein Beispiel, nämlich A = {2,3, ...,n} mit n e IN \ {0, 1}, - etwas unglücklich - "eingeflochten" ist.
>
> Dass dem so ist, erhellt daraus, dass (mal von der Endlichkeit diese Menge abgesehen)

Es ging im Beispiel von WMs besprochener Aussage explizit um endliche Mengen.

> {2,3,...,n} schon deshalb nicht als "schematischer Vertreter" für "_jede_ Menge positiver natürlicher Zahlen" stehen kann, weil nicht jede solche Menge eine 2 oder eine 3 enthalten muss (wenn sie denn überhaupt Elemente enthält).

Diese Schreibweise steht für *geordnete* Mengen.

> EOD

ROFL

Verpiss dich, Depp

Juergen Ilse

unread,
Jul 18, 2022, 10:26:14 AM7/18/22
to
Hallo,

paule32 <paule...@gmail.com> wrote:
> Am 18.07.2022 um 02:56 schrieb Juergen Ilse:
>> Das ist fuer die meisten Taschhenrechner normal, da die meisten Taschenrechner,
>> fuer die die Bezeichnung "wiseenschhhaftlich" nichht voellig zu Unrecht
>> tragen, intern eine hhoehere Rechengenauigkeit aufweisen als sie anzeigen
>> und eigentlich*immer* den Anzeigewert entsprechend runden falls noetig.
>
> ja.
> nun stell Dir aber mal vor, das sich dadurch eine Abweichung von 1 %
> ergibt - und das in kosmische Beträge ...

Wenn man eine hoehhere Genauigkeit braucht, benutzt man ein anderes
Werkzeug (sprich geeignete Software, die Recungen mit hhoeherer Genauigkeit
erlaubt) und man kalkuliert ohnehin immermit ein, dass es zu Rundungs-
fehlern kommen kann, da die genauen Werte nichht immer im Rechner
errechnet, oft nichht nicth einmal ohne zu runden dargestellt werden
koennen.

> Da wird dann ein Winkel von 1 Grad schonmal schnell 10 Grad (jetzt auf
> die Entfernung bezogen).

Nicht,wenn man zur Berechnung einen Algorithmus einsetzt, bei dem sich
Folgefehler im laufe der Rechnung eher verrigern als vergroessern.
Gewoehn dichhdran, dass man mit elektrinischen Rechenanlagen bei
Fliesskomma immer mit Rundungsfehlern rechnen muss.

> Ich weiß, das macht die Menschheit auch nicht mehr Fett.
> Aber dafür wurden ja "geeichte" Geräte entworfen.

Das hat mit "geeichhten Geraten" nicht das geringste zu tun, sondern
mit dem Einsatz passender Werkzeuge und passender Algorithmen.

Tschuess,
Juergen Ilse (jue...@usenet-verwaltung.de)

Ralf Bader

unread,
Jul 18, 2022, 11:30:45 AM7/18/22
to
On 07/17/2022 02:58 PM, Ganzhinterseher wrote:
> Drei Hinweise für Kinder, die die grundlegendsten Teile der
> Mengenlehre schon kennen.
>
> (1) Jede Menge positiver gerader natürlicher Zahlen enthält eine
> natürliche Zahl, die größer als ihre Anzahl ist. Das gilt für alle
> diese Mengen, also für unendlich viele, also auch für die unendliche
> Menge aller positiven geraden natürlichen Zahlen.
>
> (2) Auf der reellen Achse findet sich zwischen zwei beliebigen
> irrationalen Zahlen immer eine rationale Zahl. Also ist die Anzahl
> der irrationalen Zahlen nicht größer als die der rationalen Zahlen.
>
> (3) Für jede natürliche Zahl n findet sich ein Endsegment, E(n+1) =
> {n+1, n+2, n+3, ...} der natürlichen Zahlen, das n nicht enthält.
> Daraus schließen manche, dass der Schnitt aller Endsegmente leer ist.
> Sie übersehen oder verdrängen aber, dass jedes zum Beweis
> herangezogene Endsegment E(n+1) unendlich viele Zahlen enthält, so
> dass der Schnitt in keinem der unendlich viele Fälle leer ist. Für
> Fortgeschrittene: Käme in ∩{E(k) : k ∈ ℕ} = { } kein einziges
> Endsegment vor, das in ∀k ∈ ℕ: ∩{E(1), E(2), ..., E(k)} = E(k) /\
> |E(k)| = ℵ₀ (*) fehlt, so könnte der Schnitt nicht leer sein. Alle
> Endsegmente aus (*) reichen jedenfalls nicht aus, den leeren Schnitt
> zu produzieren.
>
> Gruß, WM
>

Alle 3 Ihrer Idiotismen beruhen darauf, Unendlichkeiten Eigenschaften zu
unterstellen, die nur für Endliches zutreffen. Das könnte näher
ausgeführt werden, ich verzichte darauf, da dies keine Einladung zu den
üblichen sinnfreien Diskussionen sein soll. Es ist nun so, daß
Unendliches nur abstrakt erfaßt werden kann, da es empirisch
wahrnehmbare Unendlichkeiten nicht gibt.

Der Mückenheimsche Wahn besteht darin, sich Endliches vorzustellen, das
unbegrenzt wachsen soll, daran "ganz nach hinten zu sehen", und sich
einzubilden, so ergäben sich Erkenntnisse über Unendliches. Dem ist aber
nicht so.

Mückenheim, Sie schwafeln nichts als idiotischen Scheißdreck daher, und
für Mathematik sind Sie zu doof und zu blöde. Überdies eignet Ihnen
durch Ihr Gehabe, insbesondere die penetrante Missioniererei, ein
abstoßender Charakter.

Ganzhinterseher

unread,
Jul 18, 2022, 11:42:10 AM7/18/22
to
Ralf Bader schrieb am Montag, 18. Juli 2022 um 17:30:45 UTC+2:
> Es ist nun so, daß
> Unendliches nur abstrakt erfaßt werden kann, da es empirisch
> wahrnehmbare Unendlichkeiten nicht gibt.

Aber auch im Unendlichen gelten gewisse Regeln der Mathematik Sonst hätte die ML überhaupt keinen Wert. Und ich habe nun gezeigt, dass die ML unverzichtbaren Regeln widerspricht. Dazu gehören die drei oben angewandten.
>
> Der Mückenheimsche Wahn besteht darin, sich Endliches vorzustellen, das
> unbegrenzt wachsen soll, daran "ganz nach hinten zu sehen", und sich
> einzubilden, so ergäben sich Erkenntnisse über Unendliches. Dem ist aber
> nicht so.

Für alle definierbaren Elemente ist es so. Insbesondere können Endsegmente, die
∀k ∈ ℕ-def: ∩{E(1), E(2), ..., E(k)} = E(k) /\ |E(k)| = ℵ₀
erfüllen, wenn sie zu einer Kollektion vereinigt werden, unter gar keinen Umständen einen leeren Schnitt haben. Es gilt für sie
|∩{E(k) : k ∈ ℕ_def}| = ℵ₀
Wer das bestreitet, stellt sich außerhalb der Mathematik.

Ebenso natürlich, wer behauptet, man könne durch vertauschen von X und O in
XOOOO...
XOOOO...
XOOOO...
XOOOO...
XOOOO...
...
die gesamte Matrix mit X bedecken.

Das kann behaupten wer will, und es darf glauben wer will, ich halte solche Personen für Spinner und werde das auch meinen Studenten an der Technischen Hochschule Augsburg mitteilen.

Gruß, WM

Tom Bola

unread,
Jul 18, 2022, 11:49:41 AM7/18/22
to
Ralf Bader schrieb:
Heute hat WM auf sci.math das abgesondert:

|IN| is an invariable constant

|{1, 2, 3, ...}| = | IN |

|{0, 1, 2, 3, ...}| = | IN | + 1

|{2, 4, 6, ...}| = | IN |/2

> Mückenheim, Sie schwafeln nichts als idiotischen Scheißdreck daher, und
> für Mathematik sind Sie zu doof und zu blöde. Überdies eignet Ihnen
> durch Ihr Gehabe, insbesondere die penetrante Missioniererei, ein
> abstoßender Charakter.

Ein widerliches Arschloch...

WM

unread,
Jul 18, 2022, 12:08:15 PM7/18/22
to
JVR schrieb am Sonntag, 17. Juli 2022 um 22:12:17 UTC+2:

> Also nennen sich die Fachhochschulen nun Technische Hochschulen

Nein, das ist nicht in das Ermessen einer Hochschule gestellt. Es ist eine den Leistungen entsprechend verliehene Anerkennung.

> und die Technischen
> Hochschulen nennen sich Technische Universitäten.

Durchaus nicht. ETH Zürich oder RWTH Aachen behalten ihren Titel bei.

Gruß, WM

WM

unread,
Jul 18, 2022, 12:14:22 PM7/18/22
to
Ulrich D i e z schrieb am Sonntag, 17. Juli 2022 um 23:44:47 UTC+2:
> Ganzhinterseher schrieb:
> > Drei Hinweise für Kinder, die die grundlegendsten Teile der Mengenlehre schon kennen.
> >
> > (1)
> > Jede Menge positiver gerader natürlicher Zahlen enthält eine natürliche Zahl, die größer als ihre Anzahl ist. Das gilt für alle diese Mengen, also für unendlich viele,
> Es handelt sich da um unendlich viele Mengen von denen jede die
> Eigenschaft hat, aus einer endlichen Anzahl an Elementen zu bestehen.
> > also auch für die unendliche Menge aller positiven geraden natürlichen Zahlen.
> Die jetzt in die Betrachtung einbezogene "unendliche Menge" hat diese
> Eigenschaft nicht.

Doch! Die unendliche Menge der definierbaren Zahlen ist nichts weiter als die potentiell unendliche Folge der endlichen Mengen.
>
> Woher nimmst Du die Gewissheit, dass der Schluss, es gäbe in der
> betreffenden Menge eine natürliche Zahl, die größer als ihre Anzahl ist,
> auch bei Mengen richtig ist, die aus unendlich vielen Elementen bestehen?

1) Aus der Gewissheit, dass es lediglich potentiell unendliche Mengen sind.
2) Aus der Tatsache, dass gewisse mathematische Gesetze überall gelten, wo die Mathematik nicht sinnlos wird.

> > (2)
> > Auf der reellen Achse findet sich zwischen zwei beliebigen irrationalen Zahlen immer eine rationale Zahl. Also ist die Anzahl der irrationalen Zahlen nicht größer als die der rationalen Zahlen.
> Besagte beliebige irrationale Zahlen seien der Größe nach verschieden.
> Sei A die kleinere, B die größere dieser beiden irrationalen Zahlen.
> Es befindet sich nicht nur eine rationale und auch nicht nur eine
> irrationale Zahl zwischen A und B.

Richtig. Aber man kann nicht mehr irrationale finden als rationale. Das gilt für _jede_ auffindbare Irrationalzahl. Zwei irrationale Zahlen _können_ sich gar nicht dezimal unterscheiden, ohne dass dieser Unterschied eine rationale Zahl ist. Beispiel
0,3985729485723867...
und
0,3985729485724867...
unterscheiden sich zum Beispiel durch
0,3985729485724

Und was sollten nicht dezimal unterscheidbare Irrationalzahlen sein?

> > (3)
> > Für jede natürliche Zahl n findet sich ein Endsegment, E(n+1) = {n+1, n+2, n+3, ...} der natürlichen Zahlen, das n nicht enthält. Daraus schließen manche, dass der Schnitt aller Endsegmente leer ist. Sie übersehen oder verdrängen aber, dass jedes zum Beweis herangezogene Endsegment E(n+1) unendlich viele Zahlen enthält, so dass der Schnitt in keinem der unendlich viele Fälle leer ist.
> Es reicht nicht, _irgendwelche_ Zusammenstellungen mit endlich vielen
> Endsegmenten "heranzuziehen", von denen jedes unendlich viele Elemente
> enthält, sondern man man muss, ausgehend von n=m, _alle_ Endesgmente
> E(k) für den Beweis "heranziehen", bei denen k > m ist, und das sind
> unendlich viele Endsegmente.

Alle Endsegmente, die mit ihren Vorgängern einen unendlichen Schnitt bilden, zusammengenommen, können keinen leeren Schnitt bilden.
∀k ∈ ℕ: ∩{E(1), E(2), ..., E(k)} = E(k) /\ |E(k)| = ℵ₀
|∩{E(k) : k ∈ ℕ_def}| = ℵ₀

>
> Für k >=0 enthält jedes Endsegment E(k) eine natürliche Zahl k+1, die
> die Eigenschaft hat, im Endsegment E(k+1) nicht enthalten zu sein und
> von daher auch nicht in der Schnittmenge aller Endsegmente enthalten zu
> sein. Somit hat jede natürliche Zahl die Eigenschaft, nicht in der
> Schnittmenge _aller_ Endsegmente enthalten zu sein.

Nein, denn alle diese Endsegmente enthalten unendlich viele natürliche Zahlen, also mehr als sie entfernen.

> Mengen, die Endsegmente darstellen, haben unendlich viele Elemente.

Deswegen können sie als inklusionsmonotone Mengenfolge auch keinen leeren Schnitt haben. Wo blieben denn die überall vorhandenen unendlich vielen Elemente?

> Mengen, die Endsegmente darstellen und unendlich viele Elemente haben,
> sind allesamt etwas anderes als die Schnittmenge aller dieser Mengen.

Die Schnittmenge aller definierbaren Mengen ist |∩{E(k) : k ∈ ℕ_def}| = ℵ₀ .
Beweis: Niemand kann ein Endsegment definieren, das diese Gleichung verletzt. Definierbar heißt dabei: Es gibt einen endlichen Anfangsabschnitt 1, 2, 3, ..., n, der den Index des Endsegmentes enthält.

> Die Schnittmenge aller dieser Mengen ist selbst kein Endsegment, denn
> sie ist leer,

Nicht jedenfalls die Schnittmenge aller unendlichen Endsegmente, denn sie ist nach Inklusionsmonotonie unendlich. Das ist Mathematik, die für alle Mengen gilt.

Gruß, WM

WM

unread,
Jul 18, 2022, 12:21:44 PM7/18/22
to
Ulrich D i e z schrieb am Sonntag, 17. Juli 2022 um 23:49:47 UTC+2:
> Ganzhinterseher schrieb:
> > Der Glaube an eine größere Vereinigung ist ebenso wie der Glaube an ein höheres Wesen unwissenschaftliche Privatsache. Allerdings: Die Existenz eines höheren Wesens ist nicht auszuschließen. Eine größere Vereinigung aber mit Sicherheit.
> Oft ist das Ganze mehr als nur die Summe seiner Teile. ;->

Oft, aber nicht in der Mengenlehre. Kein Element der Vereinigung aller endlichen Anfangsabschnitte darf in allen eAA fehlen. Das wird aber bei dem Sprung von allen eAA, die ja wie der Name sagt endliche sind, zum ℵo-unendlichen |N getan. In allen eAA fehlen aber ℵo Zahlen von |N.

Das ist übrigens genau das Komplement des Schnittes aller unendlichen Endsegmente. Der Rücksprung von unendlich zu leer. Sie können aber wegen Inklusionsmonotonie keinen leeren Schnitt ergeben. Solange nur ein einziges Element drin ist, ist es in allen Endsegmenten drin, nämlich eine natürliche Zahl aus E(1) = |N.

Gruß, WM

WM

unread,
Jul 18, 2022, 12:23:45 PM7/18/22
to
Fritz Feldhase schrieb am Montag, 18. Juli 2022 um 01:38:40 UTC+2:

> Welche natürliche Zahl in der Menge aller positiven geraden natürlichen Zahlen ist denn größer als die "Anzahl" der Elemente dieser Menge?
>
> Kann es so eine Zahl überhaupt geben?

Es gibt keine feste Zahl, weil es keine feste Menge gibt. Hier herrscht potentielle Unendlichkeit. Die Menge der endlichen Anfangsabschnitte definiert die definierbaren Zahlen.

Gruß, WM

WM

unread,
Jul 18, 2022, 12:29:28 PM7/18/22
to
Juergen Ilse schrieb am Montag, 18. Juli 2022 um 02:39:36 UTC+2:

> > Auf der reellen Achse findet sich zwischen zwei beliebigen irrationalen Zahlen immer eine rationale Zahl. Also ist die Anzahl der irrationalen Zahlen nicht größer als die der rationalen Zahlen.
> Flacshe Schlussfolgerung.

Zu jeder irrationalen Zahl findet man eine rationale. Das kann niemand widerlegen, weil es stimmt.

> > (3)
> > Für jede natürliche Zahl n findet sich ein Endsegment, E(n+1) = {n+1, n+2, n+3, ...} der natürlichen Zahlen, das n nicht enthält. Daraus schließen manche, dass der Schnitt aller Endsegmente leer ist.
> Diese Schlussfolgerung ist ja auch richtig.
> > Sie übersehen oder verdrängen aber, dass jedes zum Beweis herangezogene Endsegment E(n+1) unendlich viele Zahlen enthält, so dass der Schnitt in keinem der unendlich viele Fälle leer ist.
> Nein, das wird *nicht* uebersehen, weil es irrelevant ist.

Es ist relevant. Inklusionsmonotonie ist eine Grundlage der Mathematik. Alles, was ihr widerspricht, ist mathematisch wertlos.

Gruß, WM

Ulrich D i e z

unread,
Jul 18, 2022, 1:05:59 PM7/18/22
to
WM schrieb:

> Und was sollten nicht dezimal unterscheidbare Irrationalzahlen sein?

Zum Beispiel mehrere gleiche irrationale Zahlen. ;-)

Wobei sich die Frage stellt, ob der Fall, dass mehrere gleiche Zahlen
vorliegen (ersteres), überhaupt möglich ist, oder ob es sich ggfs um
mehrere Darstellungen der selben Zahl (letzteres) handelt.

(Ich kann mich noch an Diskussionen in dieser Newsgroup erinnern, in
denen manche Leute ersteres bejaht und letzteres ins Lächerliche
gezogen haben.)

Mit freundlichem Gruß

Ulrich

Fritz Feldhase

unread,
Jul 18, 2022, 1:07:10 PM7/18/22
to
On Monday, July 18, 2022 at 6:14:22 PM UTC+2, WM wrote:

> Richtig. Aber man kann nicht mehr irrationale finden als rationale.

Merkwürdig, mir erscheint es sehr leicht, mehr "zu finden":

Die rationalen Zahlen sind die Zahlen der Form n/m mit n,m e IN. Alle Zahlen der Form n/m * sqrt(2) sind irrational. Es gibt also offensichtlich mindestens so viele irrationale Zahlen wie rationale. Nun sind aber auch alle Zahlen der Form n/m * sqrt(3) irrational (wobei alle diese Zahlen von den Zahlen der Form n/m * sqrt(2) verschieden sind). Es gibt also mindestens "doppelt so viele" irrationale Zahlen wie rationale! Nun gilt aber für jede Primzahl p, dass die Zahlen der Form n/m * sqrt(p) irrational sind und sich von allen Zahlen der Form n/m * sqrt(q) - mit q prim und q =/= p - unterscheiden. Es gibt also offenbar unendlich-mal mehr irrationale Zahlen als rationale.

Gibt es all diese irrationalen Zahlen in der Mückenmatik nicht?

Fritz Feldhase

unread,
Jul 18, 2022, 1:12:12 PM7/18/22
to
On Monday, July 18, 2022 at 6:29:28 PM UTC+2, WM wrote:

> Inklusionsmonotonie ist eine Grundlage der Mathematik.

Echt jetzt? Wo wird das gelehrt?

Wird das an der technischen Hochschule Augsburg gelehrt?

> Alles, was ihr widerspricht, ist mathematisch wertlos.

Ah ja. Als GRÖMAZ musst Du es ja wissen!

Ralf Bader

unread,
Jul 18, 2022, 3:25:29 PM7/18/22
to
On 07/18/2022 05:42 PM, Ganzhinterseher wrote:

Scheißdreck

Juergen Ilse

unread,
Jul 18, 2022, 4:02:05 PM7/18/22
to
Hallo,

WM <mont...@t-online.de> wrote:
> Fritz Feldhase schrieb am Montag, 18. Juli 2022 um 01:38:40 UTC+2:
>
>> Welche natürliche Zahl in der Menge aller positiven geraden natürlichen Zahlen ist denn größer als die "Anzahl" der Elemente dieser Menge?
>>
>> Kann es so eine Zahl überhaupt geben?
>
> Es gibt keine feste Zahl, weil es keine feste Menge gibt.

Jede Menge ist "fest" und eindeutig durch ihre Elemente bestimmt.

> Hier herrscht potentielle Unendlichkeit.

Es existieren keine "nur potentiell unendlichen Mengen".
Und gaebe es in der Mathematik so etwas wie IHRE Wahnidee von
"potentiell unendlichen Kollektionen", so koennte man diesen
seltsamen Konstrukten keine Mengeneigenschaften voraussetzen,
da es keine Mengen waeren.

> Die Menge der endlichen Anfangsabschnitte definiert die definierbaren Zahlen.

Alle natuerlichen Zahhlen haben die Eigenschaften, die SIE nur den
"definierbaren" Zahlen zuordnen.

Tschuess,
Juergen Ilse (jue...@usenet-verwaltung.de)

Tom Bola

unread,
Jul 18, 2022, 4:20:11 PM7/18/22
to
Ralf Bader schrieb:

> Mückenheim, Sie schwafeln nichts als idiotischen Scheißdreck daher, und
> für Mathematik sind Sie zu doof und zu blöde. Überdies eignet Ihnen
> durch Ihr Gehabe, insbesondere die penetrante Missioniererei, ein
> abstoßender Charakter.

Wie Putin, wenn WM könnte, würde er Raketen abschiessen und Bomben werfen.

WM

unread,
Jul 19, 2022, 10:00:02 AM7/19/22
to
Ulrich D i e z schrieb am Montag, 18. Juli 2022 um 19:05:59 UTC+2:
> WM schrieb:
> > Und was sollten nicht dezimal unterscheidbare Irrationalzahlen sein?
> Zum Beispiel mehrere gleiche irrationale Zahlen. ;-)

Cantors Beweis betrifft dezimal unterscheidbare Zahlen. Dafür gilt: Jeder kann man eine eigene rationale Zahl zuordnen. Das ist offensichtlich wird von Matheologen aber abgelehnt, weil sie den Trick gebrauchen: Zuerst muss der Ungläubige seine vollständige Liste vorzeigen, und dann kommt das Diagonalargument. Müsste zuerst Cantor seine reellen Zahlen vorbringen, dann könnte man sogar für jede reelle Zahl eine eigene Liste erzeugen. Das Diagonalargument ist also nur eine unfaire Trickserei.
>
> Wobei sich die Frage stellt, ob der Fall, dass mehrere gleiche Zahlen
> vorliegen (ersteres), überhaupt möglich ist,

Jedenfalls nicht als verschiedene Punkte auf der reellen Achse.

Gruß WM

WM

unread,
Jul 19, 2022, 10:11:29 AM7/19/22
to
Fritz Feldhase schrieb am Montag, 18. Juli 2022 um 19:07:10 UTC+2:
> On Monday, July 18, 2022 at 6:14:22 PM UTC+2, WM wrote:
>
> > Richtig. Aber man kann nicht mehr irrationale finden als rationale.
> Merkwürdig, mir erscheint es sehr leicht, mehr "zu finden":

Das hängt davon ab, wer zuerst ziehen muss.
>
> Die rationalen Zahlen sind die Zahlen der Form n/m mit n,m e IN. Alle Zahlen der Form n/m * sqrt(2) sind irrational. Es gibt also offensichtlich mindestens so viele irrationale Zahlen wie rationale. Nun sind aber auch alle Zahlen der Form n/m * sqrt(3) irrational (wobei alle diese Zahlen von den Zahlen der Form n/m * sqrt(2) verschieden sind). Es gibt also mindestens "doppelt so viele" irrationale Zahlen wie rationale! Nun gilt aber für jede Primzahl p, dass die Zahlen der Form n/m * sqrt(p) irrational sind und sich von allen Zahlen der Form n/m * sqrt(q) - mit q prim und q =/= p - unterscheiden. Es gibt also offenbar unendlich-mal mehr irrationale Zahlen als rationale.

Ja, das ist ein Argument, das für definierbare Zahlen durchaus richtig ist, aber nicht viel aussagt, weil es lediglich potentiell unendliche Kollektionen betrifft. In der Mengenlehre versagt es, wegen ℵo*ℵo = ℵo.

Alle diese Argumente, angefangen von Galileis Zahlen, Quadraten oder Wurzeln sind nur für die potentiell unendlichen Kollektionen zutreffend, für die ohnehin "Bijektionen" existieren. Ich setzte Bijektionen in Anführungszeichen, weil natürlich niemals Vollständigkeit erreicht wird. Aber das Argument, dass jedes Element einen einzigen und exclusiven Partner bekommt, ist für alle potentiell unendlichen Mengen erfüllbar. Dafür ist auch Hilberts Hotel richtig.

Gruß, WM

WM

unread,
Jul 19, 2022, 10:20:03 AM7/19/22
to
Fritz Feldhase schrieb am Montag, 18. Juli 2022 um 19:12:12 UTC+2:
> On Monday, July 18, 2022 at 6:29:28 PM UTC+2, WM wrote:
>
> > Inklusionsmonotonie ist eine Grundlage der Mathematik.
> Echt jetzt? Wo wird das gelehrt?

Überall dort, wo seriöse Mathematik gelehrt wird.
>
> Wird das an der technischen Hochschule Augsburg gelehrt?

Wenn Fragen zu dem Thema aufkommen, ja.

Aber man kann auch leicht das Majorantenkriterium anwenden: Die Folge (S(k)) der Schnitte ∩{E(1), E(2), ..., E(k)} = E(k) ist für jedes unendliche Endsegment E(k) eine Majorante der Folge (n, n, n, ...) mit n ∈ ℕ. Diese Minorante hat den Grenzwert n, der also kleiner als der Grenzwert der Majorante ist. Falls die also einen Grenzwert hat, so ist es gewiss nicht die leere Menge.

> > Alles, was ihr widerspricht, ist mathematisch wertlos.
> Ah ja. Als GRÖMAZ musst Du es ja wissen!

Das hat nicht mit GRÜMAZ zu tun, sondern ist ein oft gebrauchtes Werkzeug der Analysis.

Gruß, WM

Gus Gassmann

unread,
Jul 19, 2022, 10:24:25 AM7/19/22
to
On Tuesday, 19 July 2022 at 11:20:03 UTC-3, WM wrote:
> [...] schrieb am Montag, 18. Juli 2022 um 19:12:12 UTC+2:
> > On Monday, July 18, 2022 at 6:29:28 PM UTC+2, WM wrote:
> >
> > > Inklusionsmonotonie ist eine Grundlage der Mathematik.
[...]
> Das hat nicht mit GRÜMAZ zu tun, sondern ist ein oft gebrauchtes Werkzeug der Analysis.

Anscheinend auch oft missbraucht, weil missverstanden, vor allem von GRÜMAZen überall.

Roalto

unread,
Jul 19, 2022, 11:55:32 AM7/19/22
to
Hey, Transmathematiker,
wie Ulrich sagte, der Durchschnitt aller Endsegmente ist selbst kein Endsegment, weil leer.
Ebenso, wie beliebige Durchschnitte offener Mengen nicht offen sein müssen.
Aber das passt nicht in dein Transmathematikum, weil man als Transmathematiker
nur Mathematik fühlt, aber nichts weiss.

Viel Spass weiterhin
Roalto

Ulrich D i e z

unread,
Jul 19, 2022, 5:59:42 PM7/19/22
to
Tom Bola schrieb:

> Wie Putin, wenn WM könnte, würde er Raketen abschiessen und Bomben werfen.

Was bringt dich zu dieser Annahme?

Sollte WM das Anwenden von Gewalt propagiert haben, ist mir das bisher entgangen.

Ulrich

Fritz Feldhase

unread,
Jul 19, 2022, 6:41:38 PM7/19/22
to
On Tuesday, July 19, 2022 at 11:59:42 PM UTC+2, Ulrich D i e z wrote:
> Tom Bola schrieb:
> >
> > Wie Putin, wenn WM könnte, würde er Raketen abschießen und Bomben werfen.
> >
> Was bringt dich zu dieser Annahme?
>
> Sollte WM das Anwenden von Gewalt propagiert haben, ist mir das bisher entgangen.

Stimmt. Er ist vielleicht dumm, wenn es um mathematische Belange geht, aber irgendwelche Anzeichen von Gewaltbereitschaft habe ich bei ihm auch noch nie entdeckt. Dazu ist er einfach zu entspannt in seiner Inkompetenz. :-P

Tom Bola

unread,
Jul 19, 2022, 6:50:31 PM7/19/22
to
Ulrich D i e z schrieb:
Er ist fanatisch und erklärt das Lehren der "gängigen" Mengenlehre
zu einer "schwersten Misshandlung von Schutzbefohlenen" etc.

Das und meine Menschenkenntnis reicht mir für meine Einschätzung...

Tom Bola

unread,
Jul 19, 2022, 6:50:49 PM7/19/22
to
Fritz Feldhase schrieb:

Fritz Feldhase

unread,
Jul 19, 2022, 7:01:48 PM7/19/22
to
Zur Kenntnis genommen. Ein Argument, das man wohl nicht einfach so von der Hand weisen kann. (Ich erinnere mich an diesen hirnrissigen Schwachsinn, den er da von sich gegeben hat.)

Hmmm... Nur so ein Gedanke: Kennst Du das Stanford-Prison-Experiment?

https://de.wikipedia.org/wiki/Stanford-Prison-Experiment

MfG, Fritz Feldhase

Tom Bola

unread,
Jul 19, 2022, 7:03:37 PM7/19/22
to
Fritz Feldhase schrieb:
Zb, heute sagte er (zu dir):

Ohne das Internet würde die mathematische Wahrheit weiterhin
fahrlässig missachtet oder böswillig unterdrückt werden.

Das ist reichlich militant.

Und, dasss "die Bösen" umerzogen werden müssten, sagte er ebenfalls mal,
usw, usf.

Er spricht auch manchmal viel Hass direkt aus, wenn ihm die Hutschnur
im "Dialog" platzt, aber das wird einfach übersehen von den toleranten
Matheleuten hier...

Tom Bola

unread,
Jul 19, 2022, 7:07:24 PM7/19/22
to
Fritz Feldhase schrieb:

> Zur Kenntnis genommen.

Habe gerade noch was dazu gepostet.

> ...
> Hmmm... Nur so ein Gedanke: Kennst Du das Stanford-Prison-Experiment?
>
> https://de.wikipedia.org/wiki/Stanford-Prison-Experiment

Klar. Menschen sind alles andere als eine "Konstante", gottseidank,
und doch ist wird es Leuten im allgemeinen sehr vorgeworfen, falls
die mal ihre Meinung ("plötzlich") ändern. Sicherlich, Botox-Putin
wird das wohl im wesentlichen auch nicht mehr tun...
Message has been deleted

Tom Bola

unread,
Jul 19, 2022, 8:13:54 PM7/19/22
to
Fritz Feldhase schrieb:

> On Wednesday, July 20, 2022 at 1:03:37 AM UTC+2, Tom Bola wrote:
>
>> Ohne das Internet würde die mathematische Wahrheit weiterhin
>> fahrlässig missachtet oder böswillig unterdrückt werden.
>
> Ja. Tja, klingt ein wenig nach einer "Verschwörungstheorie", um ehrlich zu sein... (bestenfalls).
>
>> Er spricht auch manchmal viel Hass direkt aus, wenn ihm die Hutschnur
>> im "Dialog" platzt, aber das wird einfach übersehen von den toleranten
>> Matheleuten hier...
>
> Ja, ok. Brauchen wir nicht weiter drüber zu diskutieren. Siehe mein anderes Post.
>
> Dennoch, muss man ihn nicht gleich mit Putin vergleichen. Ob er nun wirklich "Raketen abschießen und Bomben werfen" würde, wissen wir nicht. Vielleicht würde er sich ja auch nur mit Publikationsverboten, Hausarresten, Deportationen und kleinen feinen Morden begnügen (zumindest zu Beginn).

Ack.

Fritz Feldhase

unread,
Jul 20, 2022, 5:59:01 AM7/20/22
to
On Tuesday, July 19, 2022 at 4:11:29 PM UTC+2, WM wrote:
> Fritz Feldhase schrieb am Montag, 18. Juli 2022 um 19:07:10 UTC+2:
> > On Monday, July 18, 2022 at 6:14:22 PM UTC+2, WM wrote:
> > >
> > > Richtig. Aber man kann nicht mehr irrationale finden als rationale.
> > >
> > Merkwürdig, mir erscheint es sehr leicht, mehr "zu finden":
> >
> > Die [pos.] rationalen Zahlen sind die Zahlen der Form n/m mit n,m e IN. Alle Zahlen der Form n/m * sqrt(2) sind irrational. Es gibt also offensichtlich mindestens so viele [pos.] irrationale Zahlen wie rationale. Nun sind aber auch alle Zahlen der Form n/m * sqrt(3) irrational (wobei alle diese Zahlen von den Zahlen der Form n/m * sqrt(2) verschieden sind). Es gibt also mindestens "doppelt so viele" [pos.] irrationale Zahlen wie [pos.] rationale! Nun gilt aber für jede Primzahl p, dass die Zahlen der Form n/m * sqrt(p) irrational sind und sich von allen Zahlen der Form n/m * sqrt(q) - mit q prim und q =/= p - unterscheiden. Es gibt also offenbar unendlich-mal mehr [pos.] irrationale Zahlen als [pos.] rationale.
> >
> Ja, das ist ein Argument, das [...] durchaus richtig ist,

Jedenfalls sollte es dazu angetan sein, einen "Mittelschüler" zum Nachdenken zu bringen, dem Du Dein Ammenmärchen

"Auf der reellen Achse findet sich zwischen zwei beliebigen irrationalen Zahlen immer eine rationale Zahl. Also ist die Anzahl der irrationalen Zahlen nicht größer als die der rationalen Zahlen." [WM]

aufgetischt hast.

> [Im Kontext] der Mengenlehre versagt es [jedoch], wegen ℵo*ℵo = ℵo.

So ist es.

Wie ich schon sagte:

"All diese "Mengen" [pos.] irrationaler Zahlen sind "kammartig" ineinander "verschachtelt", wobei quasi jeweils die Zahlengerade mit allen [pos] rationalen Zahlen um den Faktor sqrt(2), sqrt(3), sqrt(5), ... nach rechts "gestreckt" ist, sodass die "Dichte" der [pos.] irrationalen Zahlen doch deutlich höher sein muss - würde man meinen - als die der [pos.] rationalen.

Nachdem man den Schüler auf diese Weise hinreichend verunsichert hat, könnte man ihm erklären, dass d a s allein aber nicht ausreicht, um auf eine größere Mächtigkeit der Menge der irrationalen Zahlen (als der der Menge der rationalen Zahlen) schließen zu können."

Hinweis: Im Kontext der Mengenlehre "versagt" Dein Ammenmärchen allerdings auch, da man hier _beweisen_ kann, dass card(I) > card(Q) ist.

Merke: Geschwafel ersetzt in der Mathematik keine Beweise.

Ganzhinterseher

unread,
Jul 20, 2022, 8:38:24 AM7/20/22
to
Roalto schrieb am Dienstag, 19. Juli 2022 um 17:55:32 UTC+2:

> wie Ulrich sagte, der Durchschnitt aller Endsegmente ist selbst kein Endsegment, weil leer.
> Ebenso, wie beliebige Durchschnitte offener Mengen nicht offen sein müssen.
> Aber das passt nicht in dein Transmathematikum, weil man als Transmathematiker
> nur Mathematik fühlt, aber nichts weiss.

Ich präsentiere hier einen streng mathematischen Beweis:

Das Minorantenkriterium wird angewandt auf die Folge (S(k)) der Beträge S(k) von Schnitten von Anfangsabschnitten unendlicher Endsegmente E(k):
S(k) = |∩{E(1), E(2), ..., E(k)}| = ℵ₀ .
Als Minorante wählen wir die konstante Folge
(M(k)) mit M(k) = 4711 für alle k ∈ ℕ .
Sie ist eine Minorante wegen 4711 < ℵ₀ .
Der Grenzwert der Minorante ist nicht größer als der Grenzwert der Folge (S(k)), falls ein solcher existiert. Und falls nicht, so ist jedenfalls 0 kein Grenzwert.

Zusammen mit der Gleichung ∩{E(k) : k ∈ ℕ} = { } beweist dies die Existenz dunkler Endsegmente.

Dieser Beweis setzt natürlich voraus, dass Endsegmente überhaupt existieren, also kurz: Cantors aktuale Unendlichkeit.

Gruß, WM


Ganzhinterseher

unread,
Jul 20, 2022, 8:47:45 AM7/20/22
to
Fritz Feldhase schrieb am Mittwoch, 20. Juli 2022 um 11:59:01 UTC+2:
>
> "All diese "Mengen" [pos.] irrationaler Zahlen sind "kammartig" ineinander "verschachtelt", wobei quasi jeweils die Zahlengerade mit allen [pos] rationalen Zahlen um den Faktor sqrt(2), sqrt(3), sqrt(5), ... nach rechts "gestreckt" ist, sodass die "Dichte" der [pos.] irrationalen Zahlen doch deutlich höher sein muss - würde man meinen - als die der [pos.] rationalen.
>
> Nachdem man den Schüler auf diese Weise hinreichend verunsichert hat, könnte man ihm erklären, dass d a s allein aber nicht ausreicht, um auf eine größere Mächtigkeit der Menge der irrationalen Zahlen (als der der Menge der rationalen Zahlen) schließen zu können."
>
> Hinweis: Im Kontext der Mengenlehre "versagt" Dein Ammenmärchen allerdings auch

nicht! Es versagt nur, wenn man Cantor das letzte Wort lässt. Gib ihm eine Liste reeller Zahlen und er zeigt eine darin nicht enthaltene reelle Zahl auf.

Zeige Deine reellen Zahlen, und ich zeige jeder einen Listenplatz à la Hilberts Hotel.

>, da man hier _beweisen_ kann, dass card(I) > card(Q) ist.

Man kann beweisen, dass card Unsinn ist, weil dunkle ganze Zahlen, Brüche und Irrationalzahlen nicht abbildbar sind. Das ist die Lösung der Unendlichkeitsparadoxien!

Gruß, WM

paule32

unread,
Jul 20, 2022, 11:06:57 AM7/20/22
to
Hallo Fritz,

> Merke: Geschwafel ersetzt in der Mathematik keine Beweise.

Merke: WM kommt von einer technischen Hochschule, und nicht
von einer anderen Denkfabrik !

Bei technischen Berechnungen geht es dadruch, das eventuell
die Statik stimmt, nicht jedoch, wie groß denn nun die Menge
C gegenüber Menge A ist.

Freilich kann man diskutieren, das für: Menge A, und B:

A = 1 Kg, und für
C = 3 kg gesetzt werden können (technisch)

das macht dan 3 Kg.

Aber betrachtet man Mengen, mit Mengen, die man nicht weiß,
wie groß sie sein können, sondern nur theoretisch, dann kommen
freilich Mehrdeutigkeiten heraus.

Mehrdeutigkeiten sind aber in der Mathematik nicht gerne
gesehen bzw. ausgeschlossen.

Beim Bau einer Brücke müssen aber eventuelle "Extreme" mit
einberechnet werden, da dann auch bestimmte gesetzliche Richt-
linien greifen ...

Aber in der Mathematik können wir nicht einfach sagen, dass
Menge (I) > Menge (Q) ist !!!

Wir Wissen es einfach nicht, wie groß die Unendlichkeiten sind
und daher muss auch die Mathematik, als Denkfabrik erkennen,
dass Mengen unterschiedliche Mächtigkeiten haben können (oder
ist schonmal jemand von uns aus dem Grab gestiegen, und hat
darüber gesprochen, wie lange er denn nun da unten in Valhalla
verbringen musste, um dann wieder als Neujahrs Barby auferstehe
konnte ?

*schorch, paule32

JVR

unread,
Jul 20, 2022, 11:38:11 AM7/20/22
to
Die Reinschrift der Behauptung ist also:
Es sei S_n, n ∈ N, eine unendliche Folge von Mengen, derart dass S_{n+1} ⊂ S_n für alle n ∈ N.
Gilt m < Card(S_n) für alle n, dann gilt auch m <= Card(lim S_n) falls der Limes existiert.

Hat sich eigentlich nie ein Student an der Hochschule Augsburg beschwert, dass der Mathelehrer
keine Ahnung von Mathe hat? Ist doch eine ziemliche Unverschämtheit. Eigentlich müsste das
strafbar sein: Wenn die zuständigen Behörden wissen konnten - und daher verpflichtet waren es
zu wissen - dass sie einem Hochstapler auf den Leim gekrochen sind und nichts dagegen unternommen
haben.

Ulrich D i e z

unread,
Jul 20, 2022, 12:06:03 PM7/20/22
to
Tom Bola schrieb:

> Ulrich D i e z schrieb:
>
>> Tom Bola schrieb:
>>
>>> Wie Putin, wenn WM könnte, würde er Raketen abschiessen und Bomben werfen.
>>
>> Was bringt dich zu dieser Annahme?
>>
>> Sollte WM das Anwenden von Gewalt propagiert haben, ist mir das bisher entgangen.
>
> Er ist fanatisch und erklärt das Lehren der "gängigen" Mengenlehre
> zu einer "schwersten Misshandlung von Schutzbefohlenen" etc.

Dazu, inwieweit das heutzutage mit Mißhandlung irgendwelcher Schutzbefohlener
einhergeht, kann ich nichts sagen, denn ich weiss nicht, welche didaktischen
Methoden zur Zeit en vogue sind.

Allerdings habe ich ganz allgenein den Eindruck, dass nicht erst heutzutage
Lehrtätigkeit häufig auch eine Misshandlung nicht nur der Schutzbefohlenen
sondern auch der Lehrkräfte darstellt, und dass das einer der Gründe für den
zunehmenden Lehrkräftemangel an deutschen Schulen sein könnte.

Schon zu meiner Schulzeit habe ich Szenarien miterlebt, bei denen die
seelische Mißhandlung von Lehrer/inne/n durch das Umfeld, was sowohl die
Schüler/innen als auch andere Lehrkräfte miteinschliesst, der Mißhandlung
mancher Schüler/innen durch das Umfeld in nichts nachstand. Das selbe später
zB auch in Seminarräumen/Praktikumssälen an der Universität.

Und so etwas ist übel. Von daher bin ich dafür, Kritik sachlich zu begründen
und Fragen sachlich zu stellen und es zu vermeiden, sich zu ergehen.

> Das und meine Menschenkenntnis reicht mir für meine Einschätzung...

Dann ist es also Spekulation und es steckt nichts dahinter, was mir entgangen
wäre.

Durch das Äussern von Spekulationen Eindrücke hervorrufen kann auch ein Unrecht
darstellen, und ggfs ein Unrecht rechtfertigt kein anderes Unrecht.

Ulrich

Ulrich D i e z

unread,
Jul 20, 2022, 12:18:39 PM7/20/22
to
Am 20.07.22 um 01:01 schrieb Fritz Feldhase:

> Hmmm... Nur so ein Gedanke: Kennst Du das Stanford-Prison-Experiment?
>
> https://de.wikipedia.org/wiki/Stanford-Prison-Experiment

Ich habe nie verstanden, aber Vermutungen, warum dieses Experiment
überhaupt durchgeführt wurde obwohl es im Lauf der Menschheitsgeschichte
mehr als genug Gefangenschaftsverhältnisse und somit mehr als genug
Datenmaterial über menschliches Verhalten und Verhaltensänderungen in
solchen Situationen gab und gibt.

Ulrich

paule32

unread,
Jul 20, 2022, 12:23:05 PM7/20/22
to
Am 20.07.2022 um 18:07 schrieb Ulrich D i e z:
> Durch das Äussern von Spekulationen Eindrücke hervorrufen kann auch ein Unrecht
> darstellen, und ggfs ein Unrecht rechtfertigt kein anderes Unrecht.

richtig !

daher: miteinander "objektiv" diskutieren, nicht pöbeln.

Das mißhandelete Lehrer oder Schüler über Jahre erdulden mussten,
das prägt schon, keine Frage: aber man muss sich auch vor Augen
halten, warum es dazu gekommen ist !

Wenn ich immer noch darüber sauer sein soll, das mich ein Kollege
um 10.000 (zehntausend) Euro betrogen hat, und es sogar zu einen
Erbschaftsstreit gekommen ward, dann hät ich bestimmt schon das
zeitliche gesegnet.

Nur was ich in all der Zeit erhofft hätte, wäre eine "kleine"
Entschuldigung, eine kleiner Satz mit vier Worten, das hätte schon
genügt...

Aber okay, die Tat ist vollbracht, die jahre gezählt, und nicht mehr
rückgängig zu machen.

Ich will nun im Jetzt, und Heute leben.
Egal, was damals war...

Vielleicht was besser machen, aber nicht grufften gehen !
Das wäre selbstmitleid, und stillstand.

Das brauch ich nicht.

paule32

Ralf Bader

unread,
Jul 20, 2022, 12:31:32 PM7/20/22
to
On 07/20/2022 02:47 PM, Ganzhinterseher wrote:
> Fritz Feldhase schrieb am Mittwoch, 20. Juli 2022 um 11:59:01 UTC+2:
>>
>> "All diese "Mengen" [pos.] irrationaler Zahlen sind "kammartig"
>> ineinander "verschachtelt", wobei quasi jeweils die Zahlengerade
>> mit allen [pos] rationalen Zahlen um den Faktor sqrt(2), sqrt(3),
>> sqrt(5), ... nach rechts "gestreckt" ist, sodass die "Dichte" der
>> [pos.] irrationalen Zahlen doch deutlich höher sein muss - würde
>> man meinen - als die der [pos.] rationalen.
>>
>> Nachdem man den Schüler auf diese Weise hinreichend verunsichert
>> hat, könnte man ihm erklären, dass d a s allein aber nicht
>> ausreicht, um auf eine größere Mächtigkeit der Menge der
>> irrationalen Zahlen (als der der Menge der rationalen Zahlen)
>> schließen zu können."
>>
>> Hinweis: Im Kontext der Mengenlehre "versagt" Dein Ammenmärchen
>> allerdings auch

Die Gleichmächtigkeit oder Gleichzahligkeit kammartig ineinander
verschachtelter Mengen ist ein BEWEISBEDÜRFTIGER Satz (bzw. muß dazu
erst einmal in korrekter Weise formuliert werden). Wenn man sich dann
den Beweis anschaut, stellt man fest, daß die Endlichkeit der
beteiligten Mengen in diesen eingegangen ist (weil der Beweis durch
Induktion über die Anzahl der Elemente erfolgt, oder darauf beruht, daß
in der kammartigen Anordnung es zu jedem enthaltenen Element (abgesehen
von den Enden) ein linkes bzw. rechtes nächtsliegendes gibt. Das muß
aber im unendlichen Fall eben nicht zutreffen (kann, wenn Wohlordnung
vorliegt, aber muß nicht)).

Auch dieser Mückenheimsche Schwachsinn beruht also darauf, Unendlichem
Eigenschaften zu unterstellen, die nur für Endliches zutreffen, und
diese durch einen verkniffenen Blick "ganz nach hinten" für begründbar
zu halten.

> nicht! Es versagt nur, wenn man Cantor das letzte Wort lässt. Gib ihm
> eine Liste reeller Zahlen und er zeigt eine darin nicht enthaltene
> reelle Zahl auf.
>
> Zeige Deine reellen Zahlen, und ich zeige jeder einen Listenplatz à
> la Hilberts Hotel.

Daß man mit der "Liste" noch andere Dinge treiben kann als in dem
Cantorschen Beweis, hat aber nicht dessen Inkorrektheit zur Folge. Das
hat man Ihnen bereits hundertemale mitgeteilt. Mückenheim, Sie sind für
Logik zu doof und zu blöde.

>> , da man hier _beweisen_ kann, dass card(I) > card(Q) ist.
>
> Man kann beweisen, dass card Unsinn ist, weil dunkle ganze Zahlen,
> Brüche und Irrationalzahlen nicht abbildbar sind. Das ist die Lösung
> der Unendlichkeitsparadoxien!

Nein, Ihr idiotischer Scheißdreck ist keine Lösung für was auch immer,
sondern einfach nur idiotischer Scheißdreck.


Ganzhinterseher

unread,
Jul 20, 2022, 1:34:25 PM7/20/22
to
JVR schrieb am Mittwoch, 20. Juli 2022 um 17:38:11 UTC+2:
> On Wednesday, July 20, 2022 at 2:38:24 PM UTC+2, Ganzhinterseher wrote:
> > Roalto schrieb am Dienstag, 19. Juli 2022 um 17:55:32 UTC+2:
> >
> > > wie Ulrich sagte, der Durchschnitt aller Endsegmente ist selbst kein Endsegment, weil leer.
> > > Ebenso, wie beliebige Durchschnitte offener Mengen nicht offen sein müssen.
> > > Aber das passt nicht in dein Transmathematikum, weil man als Transmathematiker
> > > nur Mathematik fühlt, aber nichts weiss.
> > Ich präsentiere hier einen streng mathematischen Beweis:
> >
> > Das Minorantenkriterium wird angewandt auf die Folge (S(k)) der Beträge S(k) von Schnitten von Anfangsabschnitten unendlicher Endsegmente E(k):
> > S(k) = |∩{E(1), E(2), ..., E(k)}| = ℵ₀ .
> > Als Minorante wählen wir die konstante Folge
> > (M(k)) mit M(k) = 4711 für alle k ∈ ℕ .
> > Sie ist eine Minorante wegen 4711 < ℵ₀ .
> > Der Grenzwert der Minorante ist nicht größer als der Grenzwert der Folge (S(k)), falls ein solcher existiert. Und falls nicht, so ist jedenfalls 0 kein Grenzwert.
> >
> > Zusammen mit der Gleichung ∩{E(k) : k ∈ ℕ} = { } beweist dies die Existenz dunkler Endsegmente.
> >
> > Dieser Beweis setzt natürlich voraus, dass Endsegmente überhaupt existieren, also kurz: Cantors aktuale Unendlichkeit.
> >
> Die Reinschrift der Behauptung ist also:

Die steht oben. Dein aufgeblasenes Gebaren ändert nichts an der Richtigkeit meines Argumentes.

> Es sei S_n, n ∈ N, eine unendliche Folge von Mengen, derart dass S_{n+1} ⊂ S_n für alle n ∈ N.
> Gilt m < Card(S_n) für alle n, dann gilt auch m <= Card(lim S_n) falls der Limes existiert.

So ist es. Wenn jedes Folgenglied größer als X ist dann, ist der Grenzwert nicht kleiner als X.
>
> Hat sich eigentlich nie ein Student an der Hochschule Augsburg beschwert, dass der Mathelehrer
> keine Ahnung von Mathe hat?

Das hätte sie sehr wohl tun können, wenn ein Spinner das Gegenteil gelehrt hätte. Das ist aber meines Wissens nirgendwo an der TH Augsburg geschehen.

Gruß, WM

Ganzhinterseher

unread,
Jul 20, 2022, 1:42:58 PM7/20/22
to
Ralf Bader schrieb am Mittwoch, 20. Juli 2022 um 18:31:32 UTC+2:


> Die Gleichmächtigkeit oder Gleichzahligkeit kammartig ineinander
> verschachtelter Mengen ist ein BEWEISBEDÜRFTIGER Satz

Das Cantorsche Diagonalargument ist ein falsifizierter Satz. Deswegen braucht man keinen Beweis des Gegenteils.

> > nicht! Es versagt nur, wenn man Cantor das letzte Wort lässt. Gib ihm
> > eine Liste reeller Zahlen und er zeigt eine darin nicht enthaltene
> > reelle Zahl auf.
> >
> > Zeige Deine reellen Zahlen, und ich zeige jeder einen Listenplatz à
> > la Hilberts Hotel.
> Daß man mit der "Liste" noch andere Dinge treiben kann als in dem
> Cantorschen Beweis, hat aber nicht dessen Inkorrektheit zur Folge.

Die benötigte Annahme, dass die Liste eine abgeschlossenen und nicht erweiterbare Nummerierung hat, ist falsch.

> > Man kann beweisen, dass card Unsinn ist, weil dunkle ganze Zahlen,
> > Brüche und Irrationalzahlen nicht abbildbar sind. Das ist die Lösung
> > der Unendlichkeitsparadoxien!
> Nein

Doch!

Gruß, WM

Ganzhinterseher

unread,
Jul 20, 2022, 1:49:37 PM7/20/22
to
Ulrich D i e z schrieb am Mittwoch, 20. Juli 2022 um 18:06:03 UTC+2:
>
> Allerdings habe ich ganz allgenein den Eindruck, dass nicht erst heutzutage
> Lehrtätigkeit häufig auch eine Misshandlung nicht nur der Schutzbefohlenen
> sondern auch der Lehrkräfte darstellt, und dass das einer der Gründe für den
> zunehmenden Lehrkräftemangel an deutschen Schulen sein könnte.

Dazu kann ich glücklicherweise keine negativen Erlebnisse berichten. Und wenn ich noch weiter lehren sollte, würde ich das sogar gern tun. An Unis mit linkem Gesindel, das nicht einmal ein Referat über die natürliche Bedeutung der zwei Geschlechter duldet, mag das anders sein.

Gruß, WM

Ralf Bader

unread,
Jul 20, 2022, 2:03:41 PM7/20/22
to
Eine ausführliche Darstellung aus der Feder des Veranstalters, Zimbardo,
findet sich in dessen Buch
https://www.amazon.de/Luzifer-Effekt-Macht-Umst%C3%A4nde-Psychologie-B%C3%B6sen/dp/3827419905
Die Fragestellung war, ob grausame Verhaltensweisen dem "Charakter" der
Beteiligten zuzuschreiben sind, oder durch die Situation ausgelöst
werden, in der sie sich befinden bzw. geraten sind. Vorhandenes
Datenmaterial sagt dazu nicht unbedingt viel aus. Zimbardo beschreibt
auch, wie er sich als Versuchsleiter selbst in eine Art Furor des
Experimentators verstrickte, aus dem er durch die Intervention einer
befreundeten Psychologin und späteren Ehefrau, die ohne direkte
Beteiligung die Sache als Zuschauerin verfolgte, herausgerissen wurde
und daraufhin das Experiment vorzeitig abbrach.


Ralf Bader

unread,
Jul 20, 2022, 2:06:49 PM7/20/22
to
On 07/20/2022 07:42 PM, Ganzhinterseher wrote:

Scheißdreck

JVR

unread,
Jul 20, 2022, 2:50:43 PM7/20/22
to
Sie haben immer noch nicht begriffen, dass im allgemeinen, wie auch in diesem speziellen Fall, CardLim (= 0) nicht gleich LimCard (= ℵ₀) ist.
Und ich nehme an, dass Sie das auch nicht mehr kapieren werden.
Macht aber nichts Mücke, denn das ist bei weitem nicht Ihr einziger Fehler.

paule32

unread,
Jul 20, 2022, 2:54:04 PM7/20/22
to
Am 20.07.2022 um 19:49 schrieb Ganzhinterseher:
> Dazu kann ich glücklicherweise keine negativen Erlebnisse berichten. Und wenn ich noch weiter lehren sollte, würde ich das sogar gern tun. An Unis mit linkem Gesindel, das nicht einmal ein Referat über die natürliche Bedeutung der zwei Geschlechter duldet, mag das anders sein.

ehm....
hey, mister wm....

outdäeitest Du Dich als rechter ?

Oh Mann, wie tief ist es hier gesunken das Schiff.

Pfui, ab in die Tonne.

paule32

JVR

unread,
Jul 20, 2022, 3:31:21 PM7/20/22
to
Ein bisschen deutschnational, ein wenig homophob, ein Wagner-Fan. Der weiß bestimmt,
wie das funktioniert mit den Geschlechtern - wird er uns hier aber hoffentlich nicht erklären wollen.
Was hast du von einem solchen Holzkopf anderes erwartet?

Stefan Schmitz

unread,
Jul 20, 2022, 4:17:43 PM7/20/22
to
Am 20.07.2022 um 14:47 schrieb Ganzhinterseher:

> Man kann beweisen, dass card Unsinn ist,

Wer kann das? Wo nachzulesen?

Das einzige, was du beweisen kannst, ist deine Unfähigkeit.

Hans Crauel

unread,
Jul 20, 2022, 5:12:16 PM7/20/22
to
JVR schrieb

> Hat sich eigentlich nie ein Student an der Hochschule Augsburg beschwert, dass der Mathelehrer
> keine Ahnung von Mathe hat? Ist doch eine ziemliche Unverschämtheit. Eigentlich müsste das
> strafbar sein: Wenn die zuständigen Behörden wissen konnten - und daher verpflichtet waren es
> zu wissen - dass sie einem Hochstapler auf den Leim gekrochen sind und nichts dagegen unternommen
> haben.

Die Freiheit von Lehre und Forschung ist eine hohe Hürde.
Wer erstmal an einer Hochschule verbeamtet ist, müsste
schon silberne Löffel stehlen.

Hans

Fritz Feldhase

unread,
Jul 20, 2022, 5:39:35 PM7/20/22
to
On Wednesday, July 20, 2022 at 2:47:45 PM UTC+2, Ganzhinterseher wrote:
> Fritz Feldhase schrieb am Mittwoch, 20. Juli 2022 um 11:59:01 UTC+2:
> >
> > "All diese "Mengen" [pos.] irrationaler Zahlen sind "kammartig" ineinander "verschachtelt", wobei quasi jeweils die Zahlengerade mit allen [pos] rationalen Zahlen um den Faktor sqrt(2), sqrt(3), sqrt(5), ... nach rechts "gestreckt" ist, sodass die "Dichte" der [pos.] irrationalen Zahlen doch deutlich höher sein muss - würde man meinen - als die der [pos.] rationalen.
> >
> > Nachdem man den Schüler auf diese Weise hinreichend verunsichert hat, könnte man ihm erklären, dass d a s allein aber nicht ausreicht, um auf eine größere Mächtigkeit der Menge der irrationalen Zahlen (als der der Menge der rationalen Zahlen) schließen zu können."
> >
> > Hinweis: Im Kontext der Mengenlehre "versagt" Dein Ammenmärchen allerdings[.]
> >
> Es versagt nur, wenn man Cantor das letzte Wort lässt.

Ja, genau, das sagte ich doch gerade.

> Gib ihm eine [beliebige] Liste reeller Zahlen und er zeigt eine darin nicht enthaltene reelle Zahl auf.

Ja, so was in der Art. In jeder "Liste" reeller Zahlen fehlt (mind.) eine reelle Zahl.

Im Kontext der Mengenlehre ist es jedenfalls so, dass
> >
> > man _beweisen_ kann, dass card(I) > card(Q) ist.

Simple as that.

> Man kann <saudummer Scheißdreck>

Ja, was auch immer, Mückenheim.

Gut, dass wir darüber geredet haben.

Fritz Feldhase

unread,
Jul 20, 2022, 5:59:28 PM7/20/22
to
On Wednesday, July 20, 2022 at 7:34:25 PM UTC+2, Ganzhinterseher wrote:
> JVR schrieb am Mittwoch, 20. Juli 2022 um 17:38:11 UTC+2:
> > On Wednesday, July 20, 2022 at 2:38:24 PM UTC+2, Ganzhinterseher wrote:
> > > Roalto schrieb am Dienstag, 19. Juli 2022 um 17:55:32 UTC+2:
> > >
> > > > wie Ulrich sagte, der Durchschnitt aller Endsegmente ist selbst kein Endsegment, weil leer.
> > > > Ebenso, wie beliebige Durchschnitte offener Mengen nicht offen sein müssen.
> > > > Aber das passt nicht in dein Transmathematikum, weil man als Transmathematiker
> > > > nur Mathematik fühlt, aber nichts weiss.
> > > Ich präsentiere hier einen streng mathematischen Beweis:
> > >
> > > Das Minorantenkriterium wird angewandt auf die Folge (S(k)) der Beträge S(k) von Schnitten von Anfangsabschnitten unendlicher Endsegmente E(k):
> > > S(k) = |∩{E(1), E(2), ..., E(k)}| = ℵ₀ .
> > > Als Minorante wählen wir die konstante Folge
> > > (M(k)) mit M(k) = 4711 für alle k ∈ ℕ .
> > > Sie ist eine Minorante wegen 4711 < ℵ₀ .
> > > Der Grenzwert der Minorante ist nicht größer als der Grenzwert der Folge (S(k)), falls ein solcher existiert.

Und jetzt?

> > > Zusammen mit der Gleichung ∩{E(k) : k ∈ ℕ} = { } beweist dies die Existenz dunkler Endsegmente. (*)

Ah ja. Aber (*) ist lediglich eine BEHAUPTUNG, wo ist der BEWEIS?

> > Die Reinschrift der Behauptung ist also:
> >
> > Es sei S_n, n ∈ N, eine unendliche Folge von Mengen, derart dass S_{n+1} ⊂ S_n für alle n ∈ N.
> > Gilt m < Card(S_n) für alle n, dann gilt auch m <= Card(lim S_n) falls der Limes existiert.
> >
> Wenn jedes Folgenglied größer als X ist dann, ist der Grenzwert nicht kleiner als X.

Formal: Gilt X < Card(S_n) für alle n, dann gilt auch X <= lim Card(S_n) (falls der Limes existiert).

Ihr Fehler besteht, wie üblich darin, davon auszugehen, dass lim Card(S_n) = Card(lim S_n) ist.

Offenbar haben Sie im Laufe Ihrer Ausbildung nie mitbekommen, dass man aus lim Op(a_n) nicht auf Op(lim a_n) schließen kann.

Kleines Beispiel.: Op(x) := sig(x), (a_n)_n e IN mit a_n = 1/n für alle n e IN.

lim Op(a_n) = lim sig(1/n) = lim 1 = 1.
Op(lim a_n) = sig(lim 1/n) = sig(0) = 0.

Also lim Op(a_n) =/= Op(lim a_n) in diesem Fall.

Fritz Feldhase

unread,
Jul 20, 2022, 8:43:49 PM7/20/22
to
On Wednesday, July 20, 2022 at 2:38:24 PM UTC+2, Ganzhinterseher wrote:

> Ich präsentiere hier einen streng mathematischen Beweis:

Was Du hier präsentierst, ist saudummer Scheißdreck.

> Das Minorantenkriterium wird angewandt auf die Folge (S(k)) der Beträge [? --ff] S(k) von Schnitten von Anfangsabschnitten unendlicher Endsegmente E(k):
> S(k) = |∩{E(1), E(2), ..., E(k)}| = ℵ₀ .

Hast Du das "Minorantenkriterium" auch für Folgen von Kardinalzahlen statt für Folge von reellen Zahlen (->Analysis) bewiesen?

Dazu wäre es natürlich GUT, wenn Du ZUVOR den limes-Begriff für Folgen von Kardinalzahlen definiert hättest.

Ohne so eine Definition macht jede Behauptung in Bezug auf den "Grenzwert" einer Folge von Kardinalzahlen, die keine natürlichen (und damit reelle) Zahlen sind, keinerlei Sinn.

Da ℵ₀ keine reelle Zahl ist, kann man Deinen "Beweis" schon jetzt in die Tonne treten.

Wir erkennen wieder einmal die Richtigkeit der Analyse Franz Lemmermeyers: "[WM’s] conclusions are based on the sloppiness of his notions, his inability of giving precise definitions, his fundamental misunderstanding of elementary mathematical concepts, and sometimes, as the late Dik Winter remarked [...], on nothing at all."

Juergen Ilse

unread,
Jul 21, 2022, 2:46:35 AM7/21/22
to
Ganzhinterseher <askas...@gmail.com> wrote:
> JVR schrieb am Mittwoch, 20. Juli 2022 um 17:38:11 UTC+2:
>> Die Reinschrift der Behauptung ist also:
>
>> Es sei S_n, n ∈ N, eine unendliche Folge von Mengen, derart dass S_{n+1} ⊂ S_n für alle n ∈ N.
>> Gilt m < Card(S_n) für alle n, dann gilt auch m <= Card(lim S_n) falls der Limes existiert.
>
> So ist es. Wenn jedes Folgenglied größer als X ist dann, ist der Grenzwert nicht kleiner als X.

Es ist hier ein Fehler, von "Grenzwert" zu sprechen. Denn die Mengenfolge
ist *nicht* konvergent. Der Schnitt aller Endsegmente kommt in der Folge
auch nicht vor.

Jedes Element der Mengenfolge ist der Schnitt nur *endlich* *vieler*
Endsegmente, und damit jeweils eine *unendliche* Menge. Der Schnitt
aller Endsegmente kommt da nicht vor, egal welche Folgenglieder man
betrachtet. Man kann eben nicht direkt von endlichen Kardinalzahlen
bis zur ersten unendlichen Kardinalzahl weiterzaehlen. Das geht einfach
nicht, auch wenn SIE unfaehig sind, dass zu begreifen.

Deswegen ist IHR daemlicher Versuch eines "Beweises mittels Majoranten
Kriterium" kein Beweis, sondern nur voelliger Unfug.

>> Hat sich eigentlich nie ein Student an der Hochschule Augsburg beschwert, dass der Mathelehrer
>> keine Ahnung von Mathe hat?

Vermutlich nicht. Dass man jemand wie WM eine Mathematikvorlesung vor
unschuldigen Studenten halten laesst, ist in meinnen Augen immer nochh
ein Verbrechen.

Tschuess,
Juergen Ilse (jue...@usenet-verwaltung.de)

JVR

unread,
Jul 21, 2022, 4:46:21 AM7/21/22
to
Es handelt sich hier um eine Schule, die einen 3-jährigen Kurs zur Ausbildung
von FH-Ingenieuren anbietet. Historisch haben diese Schule eine große Bedeutung
aber es sind keine Universitäten im traditionellen Sinn. Die Studenten brauchen kein
vollwertiges Abitur und es wird wenige geben, denen man nachträglich viel Mathematik
beibringen kann oder soll.
Auch wenn man diese Schulen nun 'Hochschule' nennt, ändert das nichts an der Zielsetzung.
Es gibt dort keine 'Mathematische Fakultät', auch keine Physik über das Schulniveau hinaus.
Die Lehrer sind nicht angestellt, um Forschung zu betreiben, sondern um Schüler auszubilden.
Der Mathelehrer kann nicht bestimmen, was in seiner 'Vorlesung' - in diesem Fall am ehesten
als 'Analysis-Vorkurs' beschrieben - gelehrt wird.
'Freiheit von Forschung und Lehre' an den Universitäten ist ein sehr eigenartiger Begriff,
denn gegeben hat es das nie und ob es das in der extremen Form, wo der Lehrer bestimmt,
was er lehren möchte, geben sollte, ist zweifelhaft.
Ganz sicher ist, dass der Inhalt einer Pflichtvorlesung nicht vom Lehrer bestimmt werden sollte und
dass die Studenten den Anspruch haben, dass der Lehrer von seinem Fach auch was versteht.

JVR

unread,
Jul 21, 2022, 4:55:40 AM7/21/22
to
On Thursday, July 21, 2022 at 8:46:35 AM UTC+2, Juergen Ilse wrote:
> Ganzhinterseher <askas...@gmail.com> wrote:
> > JVR schrieb am Mittwoch, 20. Juli 2022 um 17:38:11 UTC+2:
> >> Die Reinschrift der Behauptung ist also:
> >
> >> Es sei S_n, n ∈ N, eine unendliche Folge von Mengen, derart dass S_{n+1} ⊂ S_n für alle n ∈ N.
> >> Gilt m < Card(S_n) für alle n, dann gilt auch m <= Card(lim S_n) falls der Limes existiert.
> >
> > So ist es. Wenn jedes Folgenglied größer als X ist dann, ist der Grenzwert nicht kleiner als X.
> Es ist hier ein Fehler, von "Grenzwert" zu sprechen. Denn die Mengenfolge
> ist *nicht* konvergent. Der Schnitt aller Endsegmente kommt in der Folge
> auch nicht vor.

Doch, die Mengenfolge ist konvergent im üblichen Sinne, denn LimInf = LimSup.
Für abzählbare Kardinalzahlen kann man problemlos einen Limesbegriff einführen. Mücke
versteht das natürlich nicht und hat das auch nicht versucht.

Ganzhinterseher

unread,
Jul 21, 2022, 6:17:11 AM7/21/22
to
JVR schrieb am Mittwoch, 20. Juli 2022 um 20:50:43 UTC+2:

> Sie haben immer noch nicht begriffen, dass im allgemeinen, wie auch in diesem speziellen Fall, CardLim (= 0) nicht gleich LimCard (= ℵ₀) ist.

Mit Hilfe des Minorantenkriteriums beweist man, dass entweder das Minorantenkriterium oder Dein Zaubertrick des "Sprungs im Unendlichen" falsch ist. Da gibt es nichts zu "begreifen", denn das sind die einzigen Alternativen. Da das Minorantenkriterium aber in der Mathematik eine wesentliche Rolle spielt, Dein Zaubertrick hingegen außerhalb der Matheologie völlig wertlos ist, ziehe ich die mathematische Alternative vor.

Gruß, WM

Ganzhinterseher

unread,
Jul 21, 2022, 6:28:41 AM7/21/22
to
Stefan Schmitz schrieb am Mittwoch, 20. Juli 2022 um 22:17:43 UTC+2:
> Am 20.07.2022 um 14:47 schrieb Ganzhinterseher:
>
> > Man kann beweisen, dass card Unsinn ist,
> Wer kann das? Wo nachzulesen?

Na hier. Entweder akzeptierst Du den Sprung (in der Schüssel) des Rennenkampff oder die Mathematik. Dort gilt das Minorantenkriterium ausnahmslos.

Das Minorantenkriterium wird angewandt auf die Folge (S(k)) der Beträge S(k) von Schnitten von Anfangsabschnitten unendlicher Endsegmente E(k):
S(k) = |∩{E(1), E(2), ..., E(k)}| = ℵ₀ .
Als Minorante wählen wir die konstante Folge
(M(k)) mit M(k) = 4711 für alle k ∈ ℕ .
Sie ist eine Minorante wegen 4711 < ℵ₀ .
Der Grenzwert der Minorante ist nicht größer als der Grenzwert der Folge (S(k)), falls ein solcher existiert. Und falls nicht, so ist jedenfalls 0 kein Grenzwert.

Wenn Du glaubst, dass eine Folge, deren jeder Term unendlich viele Zahlen enthält, im Limes plötzlich leer sein kann, dann stehst Du außerhalb der Mathematik.

Gruß, WM
It is loading more messages.
0 new messages